Контрольно-оценочные средства по английскому языку для специальности Химическая технология неорганических веществ
Оценка 4.9

Контрольно-оценочные средства по английскому языку для специальности Химическая технология неорганических веществ

Оценка 4.9
docx
13.06.2022
Контрольно-оценочные средства по английскому языку для специальности Химическая технология неорганических веществ
КОС для 2к. ХТН Документ Microsoft Word (2).docx

Комитет образования,  науки и молодежной политики Волгоградской области

Государственное бюджетное  профессиональное  образовательное учреждение

«Волжский политехнический техникум»

 

СОГЛАСОВАНО

Методист  корпуса 1

Протокол № ___ от «___» ______ 20___ г.

_________________ Шаповалова Э.А.

 

УТВЕРЖДЕНО

на заседании ПЦК

                 гуманитарных дисциплин

Протокол № ___ от «___» ______ 20___ г.

Председатель

_________________ Дмитриева Е.Н.

 

 

СОГЛАСОВАНО

Служба стандартизации

Корпус 1

_________________ Артюшенко Н.Н.

Протокол № ___ от «___» ______ 20___ г.

 

 

 

 

 

 

Комплект контрольно-оценочных средств по учебной дисциплине

Иностранный язык

 

            Специальность18.02.03 Химическая технология неорганических веществ.

 

 

 

 

 

 

 

2022

 

 

 


Комплект оценочных средств разработан на основе рабочей программы учебной дисциплины Иностранный язык, предназначенной для выполнения требований Федерального государственного образовательного стандарта среднего профессионального образования (далее ФГОС СПО) по специальности 18.02.03 Химическая технология неорганических веществ.

 

 

 

 

 

 

 

 

 

 

 

 

 

 

 

Организация-разработчик: государственное бюджетное профессиональное образовательное учреждение «Волжский политехнический техникум» (ГБ ПОУ «ВПТ»)

 

 

Разработчик: Репникова Юлия Сергеевна.

 

Рецензенты:

Житкова Елена Сергеевна – преподаватель первой квалификационной категории  ГАПОУ «Волгоградский медико-экологический техникум».

 

Устич Наталья Алексеевна – преподаватель первой квалификационной категории ГБ ПОУ «ВПТ».

 

 

 

 

 

 

 

Содержание

 

 

стр.       

Общие положения

4

Раздел 1. Результаты освоения учебной дисциплины, подлежащие

проверке.                                  

 

4

Раздел 2 Формы контроля и оценивания по учебной дисциплине.      

 

5

Раздел 3 Оценка освоения учебной дисциплины.                  

 

6

Раздел 4 Направленность оценочных материалов на формирование

компетенций.     

 

8

 

Приложение А. Задания (типовые) для оценки освоения разделов.         13

 

Приложение Б. Задания для проверки теоретических знаний и практических умений: З1; У1; У2; У3 для рубежного контроля (по окончании 3 семестра). Задания для проверки теоретических знаний и практических умений: З1; У1; У2; У3 для итогового занятия (по окончании 4 семестра).                        74

 

 

 

 

 

 

 

 

 

 

 

 

 

 

 

 

Общие положения

Результатом освоения учебной дисциплины (УД) на втором курсе являются освоенные умения и усвоенные знания, направленные на формирование общих и профессиональных компетенций.

Проверка знаний осуществляется при помощи практических работ, которые включает в себя лексико-грамматические задания теоретической и практической направленности. При выполнении работ студенты могут пользоваться письменными принадлежностями, Google переводчиком.

Итоговая проверка осуществляется при помощи итоговых контрольных работ по окончании 3 и 4 семестра, которые включает в себя десять лексико-грамматических заданий теоретической и практической направленности. При выполнении итоговых контрольных работ обучающиеся могут пользоваться письменными принадлежностями, учебниками, словарями.

На выполнение каждой итоговой контрольной работы отводится 60 мин.

Комплект оценочных средств раскрывает содержание и требования к текущему контролю по УД.

Раздел 1. Результаты освоения учебной дисциплины, подлежащие проверке.

В результате контроля и оценки по учебной дисциплине осуществляется комплексная проверка следующихобразовательных результатов:

В результате освоения учебной дисциплины студент должен уметь:

У1 - общаться (устно и письменно) на иностранном языке на профессиональные и повседневные темы;

У2 - переводить (со словарем) иностранные тексты профессиональной направленности;

У3 - самостоятельно совершенствовать устную и письменную речь, пополнять словарный запас.

В результате освоения учебной дисциплины студент должен знать:

 

З1 -  лексический (1200-1400 лексических единиц) и грамматический минимум, необходимый для чтения и перевода (со словарем) иностранных текстов профессиональной направленности.

Раздел 2 Формы текущего контроля и оценивания по учебной дисциплине.

Раздел / Тема учебной дисциплины

Форма текущего контроля и оценивания

Раздел 1.Основной раздел

Тема 1.1. Образование в России и за рубежом, среднее профессиональное образование. Глаголы в действительном залоге, в Indefinite, Progressive, Perfect (в простых, продолженных и совершенных временах).

 

Тема 1.2. Государственное устройство, правовые институты России и англоязычных стран. Формы страдательного залога английских глаголов.

 

 

Тема 1.3 Навыки общественной жизни (повседневное поведение, профессиональные навыки и умения).Модальные глаголы и их эквиваленты.

 

 

 

 

Тема 1.4 Здоровый образ жизни.Сложноподчинённые предложения с союзами for, as, till, until, (as) though.

 

 

 

Тема 1.5 Профессии, карьера.Сложносочинённые предложения. Сложноподчинённые предложения.Типы придаточных  предложений.

 

 

Тема 1.6 Отдых, каникулы, отпуск. Туризм. Вспомогательные глаголы to be, to have, to do в английском языке.

 

 

Тема 1.7 Молодежные течения. Виды развлечений современной молодёжи.

 Сослагательное и повелительное наклонение в английском языке.

 

 

Раздел 2. Профессионально-направленный раздел

Тема 2.1 Цифры, числа, математические действия.Условные предложения трёх типов.

 

 

 

Тема 2.2 Основные геометрические понятия и физические явления. Инфинитив, его формы, инфинитивные обороты.

 

 

 

Тема 2.3 Промышленность, транспорт; детали, механизмы. Герундий и герундиальные обороты.

 

 

 

Тема 2.4 Оборудование, работа. Причастие, его формы, отличие причастия от герундия.

 

 

 

Проверка полученных знаний путем устного и письменного опросов, тестирования и их оценка, наблюдение за ходом выполнения практических работ и их оценка.

 

Проверка полученных знаний путем устного и письменного опросов, тестирования и их оценка, наблюдение за ходом выполнения практических работ и их оценка.

 

Проверка полученных знаний путем устного и письменного опросов, тестирования и их оценка, наблюдение за ходом выполнения практических работ и их оценка.

 

 

Проверка полученных знаний путем устного и письменного опросов, тестирования и их оценка, наблюдение за ходом выполнения практических работ и их оценка.

 

Проверка полученных знаний путем устного и письменного опросов, тестирования и их оценка, наблюдение за ходом выполнения практических работ и их оценка.

Проверка полученных знаний путем устного и письменного опросов, тестирования и их оценка, наблюдение за ходом выполнения практических работ и их оценка.

Проверка полученных знаний путем устного и письменного опросов, тестирования и их оценка, наблюдение за ходом выполнения практических работ и их оценка.

 

Проверка полученных знаний путем устного и письменного опросов, тестирования и их оценка, наблюдение за ходом выполнения практических работ и их оценка.

 

Проверка полученных знаний путем устного и письменного опросов, тестирования и их оценка, наблюдение за ходом выполнения практических работ и их оценка.

 

Проверка полученных знаний путем устного и письменного опросов, тестирования и их оценка, наблюдение за ходом выполнения практических работ и их оценка.

Проверка полученных знаний путем устного и письменного опросов, тестирования и их оценка, наблюдение за ходом выполнения практических работ и их оценка.

 

Раздел 3 Оценка освоения учебной дисциплины

3.1 Перечень заданий и вопросов для оценки освоения разделов/тем учебной дисциплины

Раздел 1. Основной

Тема 1.1Образование в России и за рубежом, среднее профессиональное образование. Глаголы в действительном залоге, в Indefinite, Progressive, Perfect (в простых, продолженных и совершенных временах).

Проверяемые результаты обучения темы 1.1: (У1; У2; У3; З1)

- перечень вопросов теоретического характера:

1. Проанализировать и охарактеризовать особенности образования в России и за рубежом, специфику среднего профессионального образования.

2. Охарактеризовать образование действительного залога в простых временах в утвердительных, вопросительных и отрицательных предложениях.

3. Охарактеризовать образование действительного залога впродолженных и совершенныхвременах в утвердительных, вопросительных и отрицательных предложениях.

Перечень практических заданий, освещающих аспекты темы 1.1  Приложение А.

Тема 1.2 Государственное устройство, правовые институты России и англоязычных стран. Формы страдательного залога английских глаголов.

Проверяемые результаты обучения темы 1.2: (У1; У2; У3; З1)

- перечень вопросов теоретического характера:

1. Охарактеризовать государственное устройство, правовые институты России и англоязычных стран.

2.Обозначитьформы страдательного залога английских глаголов в простых временах.

3. Обозначить формы страдательного залога английских глаголов впродолженных и совершенных временах.

Перечень практических заданий, освещающих аспекты темы 1.2 Приложение А.

Тема 1.3 Навыки общественной жизни (повседневное поведение, профессиональные навыки и умения). Модальные глаголы и их эквиваленты.

Проверяемые результаты обучения темы 1.3: (У1; У2; У3; З1)

- перечень вопросов теоретического характера:

1.Охарактеризовать свой типичный рабочий день.

2. Охарактеризовать жёсткие и гибкие профессиональные навыки.

 

3. Назвать модальные глаголы и их эквиваленты, их специфику, случаи употребления.

Перечень практических заданий, освещающих аспекты темы 1.3 Приложение А.

Тема 1.4 Здоровый образ жизни. Сложноподчинённые предложения с союзами for, as, till, until, (as) though.

Проверяемые результаты обучения темы 1.4: (У1; У2; У3; З1)

- перечень вопросов теоретического характера:

1. Охарактеризовать принципы, главные составляющие здорового образа жизни.

2. Насколько важен спорт в жизни современного человека?

3. Чем характеризуются сложноподчинённые предложения с союзами for, as, till, until, (as) though?

Перечень практических заданий, освещающих аспекты темы 1.4 Приложение А.

Тема 1.5 Профессии, карьера. Сложносочинённые предложения. Сложноподчинённые предложения. Типы придаточных  предложений.

Проверяемые результаты обучения темы 1.5: (У1; У2; У3; З1)

- перечень вопросов теоретического характера:

1. Проблема планирования карьеры, профессионального самоопределения.

2. Специфика сложносочинённых предложений.

3.Специфика сложноподчинённых предложений, типов придаточных  предложений.

Перечень практических заданий, освещающих аспекты темы 1.5 Приложение А.

Тема 1.6 Отдых, каникулы, отпуск. Туризм. Вспомогательные глаголы to be, to have, to do в английском языке.

Проверяемые результаты обучения темы 1.6: (У1; У2; У3; З1)

- перечень вопросов теоретического характера:

1. Каковы преимущества и недостатки путешествий с помощью различных видов транспорта?

2. Какие существуют виды туризма?

3. Каковы особенности и случаи употребления вспомогательных глаголов to be, to have, to do в английском языке, их формы в различных временах?

       Перечень практических заданий, освещающих аспекты темы 1.6  Приложение А.

Тема 1.7 Молодежные течения. Проблемы молодежи. Виды развлечений современной молодёжи. Сослагательное и повелительное наклонение в английском языке.

Проверяемые результаты обучения темы 1.6: (У1; У2; У3; З1)

- перечень вопросов теоретического характера:

1. Каковы основные проблемы молодежи?

2. Какие существуют молодежные течения?

3. Каковы особенности сослагательного и повелительного наклонения в английском языке?

       Перечень практических заданий, освещающих аспекты темы 1.7  Приложение А.

Раздел 2. Профессионально-направленный раздел

Тема 2.1 Цифры, числа, математические действия. Условные предложения трёх типов.

Проверяемые результаты обучения темы 2.1: (У1; У2; У3; З1)

- перечень вопросов теоретического характера:

1. Назвать цифры, числа, (количественные и порядковые числительные), математические действия на английском языке.

2.Охарактеризовать условные предложения трёх типов.

3. Охарактеризовать бессоюзные условные предложения.

Перечень практических заданий, освещающих аспекты темы 2.1  Приложение А.

Тема 2.2 Основные геометрические понятия и физические явления. Инфинитив, его формы, инфинитивные обороты.

Проверяемые результаты обучения темы 2.1: (У1; У2; У3; З1)

- перечень вопросов теоретического характера:

1. Назвать основные геометрические понятия и физические явления на английском языке.

2. Употребление инфинитива с частицей to и без нее.

3. Особенности инфинитивных оборотов Сложное дополнение и Сложное подлежащее.

Перечень практических заданий, освещающих аспекты темы 2.2  Приложение А.

Тема 2.3 Промышленность, транспорт; детали, механизмы. Герундий и герундиальные обороты.

Проверяемые результаты обучения темы 2.3: (У1; У2; У3; З1)

- перечень вопросов теоретического характера:

1. Назвать виды промышленности, их специфику на английском языке.

2. Особенности герундия, его формы, варианты перевода, функции в предложении.

3. Особенности герундиальных оборотов, способы перевода.

4. Характеристика транспорта будущего.

Перечень практических заданий, освещающих аспекты темы 2.3 Приложение А.

Тема 2.4 Оборудование, работа. Причастие, его формы, отличие причастия от герундия. Причастные обороты.

Проверяемые результаты обучения темы 2.4: (У1; У2; У3; З1)

- перечень вопросов теоретического характера:

1. Назвать виды оборудования, их специфику на английском языке.

2. Охарактеризовать причастие, его формы.

3. Чем причастие отличается от герундия?

4. Отличие зависимых причастных оборотов от абсолютных независимых причастных оборотов.

Перечень практических заданий, освещающих аспекты темы 2.4 Приложение А.

Задания для проверки теоретических знаний и практических умений: З1; У1; У2; У3 контрольной работы, Приложение B.

Раздел 4 Направленность проверяемых умений и знаний на ПК и ОК

Коды компетенций (ПК, ОК)

Коды проверяемых У и З

ПК 1.1Использовать английский язык для общения в профилирующей области науки и для делового профессионального общения.

У1; У2; У3; З1

ОК 1. Понимать сущность и социальную значимость своей будущей профессии, проявлять к ней устойчивый интерес.

У1; У2; У3; З1

ОК 2. Организовывать собственную деятельность, выбирать типовые методы и способы выполнения профессиональных задач, оценивать их эффективность и качество.

У1; У2; У3; З1

ОК 3. Принимать решения в стандартных и нести за них ответственность.

У1; У2; У3; З1

ОК 4. Осуществление поиска и использование информации, необходимой для эффективного выполнения профессиональных задач, профессионального и личностного развития.

У1; У2; У3; З1

ОК 5. Использовать информационно-коммуникационные технологии в профессиональной деятельности.

У1; У2; У3; З1

ОК 6. Работать в коллективе и в команде, эффективно общаться с коллегами, руководством, потребителями.

У1; У2; У3; З1

ОК.7 Брать на себя ответственность за работу членов команды (подчинённых), за результат выполнения заданий.

У1; У2; У3; З1

ОК 8. Самостоятельно определять задачи профессионального и личностного развития, заниматься самообразованием, осознанно планировать повышение квалификации.

У1; У2; У3; З1

ОК 9. Ориентироваться в условиях частой смены технологий в профессиональной деятельности.

У1; У2; У3; З1

 

 

 

 

 

 

 

 

 

 

 

 

 

 

 

 

 

Приложение А (обязательное).  Задания (типовые) для оценки освоения разделов.        

Раздел 1. Тема 1.1Образование в России и за рубежом, среднее профессиональное образование. Глаголы в действительном залоге, в Indefinite, Progressive, Perfect Tenses (в простых, продолженных и совершенных временах).

Проверяемые результаты обучения: владение навыками чтения и перевода, знание форм глагола в простых, продолженных и совершенных временах), умение распознавать и употреблять их в речи.

Задания (типовые) для оценки освоения раздела 1, темы 1.1Образование в России и за рубежом, среднее профессиональное образование. Глаголы в действительном залоге, в Indefinite, Progressive, Perfect Tenses (в простых, продолженных и совершенных временах).

Задание 1. Ознакомьтесь с лексикой по теме Образование. Прочитайте тексты про образование в России и Великобритании, сделайте краткое изложение содержания каждого текста на английском и русском языках, ответьте на вопросы к текстам письменно.

EDUCATION IN RUSSIA

People in our country have the right for education. It is our Constitutional right. But it is not only a right, it is a duty, too. Every boy and every girl in Russia must go to school, that is, they must get a full secondary education. So, when they are 6 or 7 years old they begin to go to school. There are thousands of schools in Russia.

There are schools of general education, where the pupils study Russian (or a native language), Literature, Mathematics, History, Biology, Music, Arts, Foreign Languages.

There is also a number of specialized schools, where the pupils get deep knowledge of foreign languages, or Maths, or Physics. After finishing 9 classes of secondary school young people can continue their education at different kinds of vocational or technical schools or colleges. They not only learn general subjects, but receive a speciality there. Having finished a secondary school, a technical school or a college young people can start working, or they may enter an Institute or a University. Professional training makes it easier to get higher education. As for high schools, there are a lot of them in our country. Some of them train teachers, others — doctors, engineers, architects, actors and so on.

Many institutes have evening and extra-mural departments. That gives the students an opportunity to study at an institute without leaving their jobs.

Words:

right                     право

duty                     обязанность

secondary             среднее

deep                     глубокий

vocational school профтехучилище

general                 общий

to receive              получать

training                обучение

higher                   высшее

extra-mural          заочный

opportunity          возможность

Questions:

1. Is education in our country free?

2. Is education in Russia right or duty?

3. What kind of schools are there in Russia?

4. What are the possible ways to continue education after the finishing of the secondary school?

5. What are the main types of educational institutions in our country?

6. What are the types of higher education institutions in Russia?

THE SYSTEM OF EDUCATION IN GREAT BRITAIN

The system of education in any country is aimed at developing a personality for the good of the individual and society as a whole. Pre-school education in England begins at the age of 3 or 4. Around half of the children at this age attend nursery schools or playgroups mostly organized by parents. Children of this age need care as well as education. That's why kids play a lot, learn to listen attentively and to behave.

Compulsory primary education begins at the age of five in England, Wales and Scotland and at four in Northern Ireland. Children start their school career in an infant school. Lessons start at 9 a. m. and are over at 4 p. m. They are taught «3 R's»: Reading, wRiting, aRithtnetic. Pupils have a lot of fun at school, drawing, reading, dancing or singing. When they are 7 pupils move to a junior school, which lasts four years till they are 11. They study a lot of subjects: English, Mathematics, Science, History, Geography along with Technology, Music, Art and Physical education Most of children (over 90 per cent) go to state schools where education is free. Only a small proportion of them attend private (Public) or independent schools. Parents have to pay for the education at these schools. The fees are high and only some families can afford it. So such schools are for the representatives of the high class. The most notable Public schools are Eton, Harrow, Winchester, Rugby.

Secondary education begins at 11. The majority of secondary schools are Comprehensive schools where boys and girls study together. Besides, parents can take their sons and daughters to Grammar schools or Secondary Modern schools. Grammar schools provide an academic course from 11 to 18. They prepare pupils for colleges and universities.

Many children of working class families go to Modern schools. They give a very limited education. Pupils get instruction in woodwork, metalwork, sewing, shorthand, typing and cooking. After finishing such a school a pupil becomes an unskilled worker. The Comprehensive Schools have their own «Grammar school* classes and *Modern classes*.  Every pupil has to choose a set of subjects to learn. If he takes up Art he will study English Literature, Music, Art, Drama and foreign languages. If he is good at exact and natural sciences, he will learn Science: Mathematics, Physics, Chemistry, Biology, Geography, Economics and Technical Drawing.

The British government encourages careers education in the country. That's why secondary schools try to break down the barriers between education and business. They set up close links with firms to allow their students to take part in business activities. At around 16 years old teenagers take some exams and coursework to get General Certificate of Education. Those who choose to stay on at school usually study for two further years to pass A level (Advanced level) exams. These exams will give them a chance to enter the university.

Words:

to be aimed to — преследоватьцель

рге-school — дошкольное

to attend — посещать

compulsory — обязательный

primary education — начальное образование

infant school, nursery school — подготовительная школа, детский сад junior school — начальная школа

science — естествознание

secondary education — среднее образование

limited — ограниченный

sewing — шитье

shorthand — стенография

unskilled — неквалифицированный

to encourage — поощрять

link — связь

Questions:

1. What is a system of education aimed to?

2. When does the pre-school education in England begin in England, Wales and North Ireland?

3. When does the compulsory education begin in England?

4. What are «3R's» of the infant school?

5. What are the most famous Public schools in England?

6. What are Grammar and Comprehensive schools?

7. What are Modern schools?

8. Are there compulsory subjects in UK?

9. What exams must be taken to enter the University?

Задание 2. Divide the following words into three groups, those which describe a) informal education, b) formal education, c) special education. Распределите слова по группам:a) неформальное образование, b) формальное образование, c) специальное образование.

Library, museum, a teacher, schools, colleges, universities, a videotape, a television show, informal manner, to pass exams, a certificate, a diploma, a degree, gifted children, adults, general education, vocational education.

Задание 3. Make up phrases by matching words from two columns. Составьте фразы по теме образование, соединив по смыслу прилагательные с существительными из колонок.

Н-р:well-informed citizens

general                                    students

Early                                       understanding

gifted                                      scheme

Vocational                              childhood

own                                        education

Useful                                     children

cultural                                   training

Technical school                      heritage

secondary                               initiative

practical                                  schools

Задание 4. Match each word with its correct definition. Соотнесите слово по теме Образование с его определением.

Carpentry, to transmit, experience, to gain, intelligent, a skill, a sense, a manner, an adult, a heritage

1)                a special ability to do something well, gained by learning and practice

2)                the way or method in which something is done or happens

3)                good and especially practical understanding

4)                an object, custom, or quality which is passed down over many years within a nation, social group, or  family

5)                the art of work  of a person who is skilled at making and repairing wooden objects, especially one who does this as a job

6)                a fully grown person, usually 18 or 21

7)                having or showing powers of learning, reasoning, or understanding

8)                to send or pass from one person, place, thing to another

9)                knowledge or skill which comes from practice in an activity or doing something for a long time

10)           to get something useful, wanted

Задание 5.  Fill in the gaps with the missing words. Заполните пропуски в предложениях подходящими по смыслу словами по теме Образование.

1)                We consider _________to be the ways in which people learn skills, gain_________ and understanding about the world and about themselves.

2)                _________ expect learners to come to school regularly and on time.

3)                Teachers want learners to pass_________ successfully.

4)                Teachers want them to take up their education after leaving_________.

5)                At secondary school pupils are taught different _________.

Задание 6. Поставьте предложения с глаголами в простых временах в действительном залоге в вопросительную и отрицательную формы.

1. Не studies at the college. 2. They play football well. 3. We usually watched TV in the evening. 4. Our teacher asks many questions. 5. Nick worked at school last year. 6. We shall go to St. Petersburg in summer. 7. They went to Moscow. 8. He will visit us some day. 9. They study English. 10. My sister finished her work. Задание 7. Поставьте глаголы,  данные в скобках, в  нужную форму, укажите время глагола.

1. Peter and Ann (go) away five minute s ago. 2. I (write) the letter but I (not send) it. 3. He just (go) away. 4. She already (answer) the letter. 5. She (answer) it on Tuesday. 6.1 just (tell) you the answer. 7. I (read) that book in my summer holidays. 8. 1 (not see) him for three years. I (be) glad to see him again some time. 9. What you (do)? — I (copy) the text from the textbook now. 10. He (go) to Moscow next week? 22. Here is your watch. I just (find) it. 23. You (nothave) yourbreakfastyet?

Задание 8. Переведите предложения на английский язык, обращая внимание на форму глагола сказуемого.

1. Я никогда об этом не слышал. 2. Я только что прочитал ваше письмо. 3. Вы уже купили новую квартиру? 4. Вы сделали много ошибок в диктанте. 5. Вы когда-нибудь видели этого человека? 6. В этом месяце я прочитал две новых книги. 7. Мой друг уехал в Москву неделю назад и еще не писал мне. 8. Я не видел новых фильмов за последнее время (lately). 9. Вы читали сегодня в газете о нашем новом театре? 10. Вы были когда-нибудь в Лондоне? — Нет, я поеду туда в этом году.

Задание 9. Преобразуйте следующие предложения в вопросительно-отрицательные и дайте краткие ответы.

Образец: They are at home. — Aren't they at home? — Yes, they are. — No, they aren't. Они дома. — Разве они не дома? — Да, дома. — Нет, не дома.

1. They left for Moscow. 2. He has finished his work. 3. She will visit us on Sunday. 4. She has many relatives. 5. His father works here. 6. You know his address. 7. We shall go home together. 8. He can play chess. 9. I am listening to you. 10. His friends were playing football. 11. You have done the task.

Критерии оценки:

5 «отлично» -  Правильное выполнение заданий. Студент отлично знает и применяет на практике грамматический материал по теме Глаголы в действительном залоге, в Indefinite, Progressive, Perfect Tenses (в простых, продолженных и совершенных временах). Студент владеет необходимой  лексикой по теме Образование в России и за рубежом, среднее профессиональное образование, обладает навыками чтения и литературного перевода.

4 «хорошо» - Правильное выполнение 8 заданий из 9. Студент хорошо знает и применяет на практике грамматический материал по теме Глаголы в действительном залоге, в Indefinite, Progressive, Perfect Tenses(в простых, продолженных и совершенных временах). Студент владеет необходимой  лексикой по теме Образование в России и за рубежом, среднее профессиональное образование, обладает навыками чтения и литературного перевода. Допускаются неточности при выполнении заданий.

3 «удовлетворительно» - Допускаются неточности и незначительные ошибки в выполнении  заданий. Студент не достаточно хорошо знает и применяет на практике грамматический материал по теме Глаголы в действительном залоге, в Indefinite, Progressive, Perfect Tenses(в простых, продолженных и совершенных временах). Студент не в полной мере владеет необходимой  лексикой по теме Образование в России и за рубежом, среднее профессиональное образование, обладает навыками чтения и литературного перевода.Объем правильно выполненных заданий составляет 6заданий.

2 «неудовлетворительно» - Отсутствует выполнение заданий. Студент не знает и не применяет на практике грамматический материал по теме Глаголы в действительном залоге, в Indefinite, Progressive, Perfect Tenses(в простых, продолженных и совершенных временах). Студент не владеет необходимой  лексикой по темеОбразование в России и за рубежом, среднее профессиональное образование, не обладает навыками чтения и литературного перевода. Объем правильно выполненных заданий составляет менее 4 заданий из 9.

 

Раздел 1. Тема 1.2 Государственное устройство, правовые институты России и англоязычных стран. Страдательный залог английского глагола.

Проверяемые результаты обучения: владение навыками чтения и перевода, знание формстрадательного залога, умение ставить глаголы в требуемые формы страдательного залога, употреблять страдательный залог в устной речи.

Задания (типовые) для оценки освоения раздела 1, темы 1.2 Государственное устройство, правовые институты России и англоязычных стран. Страдательный залог английского глагола.

Задание 1.Запишите слова с транскрипцией.

legislative – ['leʤɪslətɪv] – законодательный 
executive –  [ɪg'zekjutɪv] – исполнительный 
judicial –  [ʤu:'dɪʃ(ə)l] – судебный
branch –  [brɑ:nʧ] – ветвь 
to vote –  [vəut] – голосовать 
to elect –  [ɪ'lekt] –  избирать
government – ['gʌv(ə)nmənt] – правительство
to guarantee ['gærən'ti:] гарантировать
to dissolve [dɪ'sÁlv] распускать
a council ['kaunsl] совет
an assembly [ə'semblɪ] собрание
basic ['beɪsɪk] основной
а chairman ['t∫eəmən] председатель
a deputy ['depjutɪ] депутат
Russia [ˈrʌʃə] Россия
Political system [pəˈlɪtɪk(ə)l ˈsɪstəm] Политическая система
the head of statе [ðiː hed ɒvsteɪt] глава государства
the President [ðiːˈprɛzɪd(ə)nt]  Президент
the Federa lAssembly [ðiː ˈfɛd(ə)r(ə)l  əˈsem.bli] Федеральное собрание
the Federal Government   [ðiːˈfɛd(ə)r(ə)l  ˈɡʌv(ə)nˌm(ə)nt] Федеральное правительство
the Duma  [ðiːˈduːmə] Дума
the Constitutional Court [ðiː kɒnstɪˈtjuːʃ(ə)n(ə)l kɔːt] Конституционный Суд

Задание 2. Ознакомьтесь с содержанием текста, ответьте на вопросы письменно.

What Political System Does Russia Belong To?

The Russian Federation (Russia) is a presidential republic. The President is the head of state and is elected directly by the people. In fact he has much power. The President can even dissolve the Duma if it doesn’t agree with his suggestions three times running. The President has his Administration but it is not part of the Federal Government. The President is involved in the work of the legislative and executive branches.

The Federal Assembly represents the legislative branch of power. It is made up of two houses: the Federation Council and the Duma which make laws. The President can veto laws passed by the Federal Assembly but the Federal Assembly can pass over the President’s veto by a two-thirds majority.

The Federal Government represents the executive branch of power. The President appoints its head, the Chairman of the Government, but the Duma must approve his appointment.

The Supreme Court represents the judicial branch of power.

The Constitutional Court has the right to declare action of the President, the Federal Assembly and the Federal Government unconstitutional.

1) What branches does the country government consist of?
2) What does the judicial  branch of power consist of?
3) Whom does legislative branch belong to?
4) Whom does executive branch belong to?
5) By whom the President is elected by?
6) Is Russia presidential republic or federal republic?
7) What branch of power does the Federal assembly represent?
8) Which branch of power does the Federation Council belong to?
9) What does the Federal Government consist of?
10) Who approves ministers?
11) What is the role of the Supreme Court?
12) What is the role of the Constitutional Court?
13) The people. What is the people’s role in the political system?

Задание 3. Прочитайте тексты, сравните политическое устройство Британии и США, дайте их краткую характеристику на английском и русском языках.

Government in the UK

The centre of the national government in the UK is Parliament, which makes all the important laws for the country. Parliament is made up of the House of Commons, the House of Lords, and the Queen or King. The Houses of Parliament are in a part of London called Westminster, and the word Westminster is often used to mean Parliament. The House of Commons is more powerful than the House of Lords. It has 650 members, called Members of Parliament or MPs, each representing a constituency (an area of land and the people who live in it). The House of Lords has over 1000 members.

The government brings bills (suggested new laws) to the House of Commons, where they are discussed by MPs. The bills then go to the House of Lords. The House of Lords can suggest changes to a bill, but does not have the power to stop it from becoming law. When the bills come back to the Commons, MPs vote on them, and if they are passed they are signed by the Queen or King and become Acts of Parliament, which means that they become part of British law.

In 1997, the people of Scotland and Wales voted for devolution, which means that they will have their own separate parliaments, known as the Scottish Parliament or the Scottish Assembly, and the Welsh Parliament or the Welsh Assembly. Westminster will continue to deal with things such as foreign affairs, defence and immigration, and the Scottish and Welsh parliaments will deal with things such as health, education, transport, and the environment.

The organizations that are responsible for local government in the UK are called councils. Their main job is to provide local service such as schools, libraries, and the Fire Service. They are also responsible for the local environment, and take rubbish from people’s houses and clean the streets.

Government in the US

There are three levels of government in the US — federal, state, and local. All of these are elected by the people of the country.

The federal government is the national government of the US. The Constitution of the US limits the power of the federal government to defence, foreign affairs, printing money, controlling trade and relations between the states, and protecting human rights. The federal government is made up of Congress, the President, and the Supreme Court.

Congress is the institution that makes laws, and is made up of the House of Representatives and the Senate. The House of Representatives has 435 members called Representatives or Congressmen and Congresswomen, who are elected by the people of a state to represent that state. The number of Representatives for each state depends on the size of the population of the state. The Senate has 100 members called Senators, who are elected by their state. Each state has two Senators.

State government has the greatest influence over people’s daily lives. Each state has its own written constitution (set of fixed laws). There are sometimes great differences in law between the different states, concerning things such as property, crime, health, and education. The highest elected official of each state is the Governor.

The organizations that are responsible for local government in the US are called town or city or county councils. They make laws which concern things such as traffic, when and where alcohol can be sold, and keeping animals. The highest elected official of a town or city is usually the Major.

Every law at every level of government must be in agreement with the United States Constitution.

Задание 4. Переведите и определите залог и время сказуемого.

1.They can be seen in our library every day. 2. The delegation was met at the airport. 3. The child was often left home alone. 4. These houses were built last year. 5. All letters had been written when we came. 6. This film is much spoken about. 7. The machine is being tested now. 8. His work has been already finished. 9. I was told to wait in the reception room. 10. Your letter will have been answered by Monday.

Задание 5. Раскройте скобки, поставьте глаголы в нужные формы страдательного залога.

1. I'm not reading these books today. They (return) to the library. 2. The paintings (exhibit) till the end of the month. 3. Why your home task (not do)? 4. The patient (take) to the hospital today, and (operate) tomorrow morning. 5. This room (use) only on special occasions. 6. Bicycles must not (leave) here. 7. This newspaper (not read) because the pages (not cut). 8. Dictionaries may not (use) at the examination. 9. Usually the floor (sweep) every day, but it (not sweep) yesterday. 10. This book (leave) in the classroom yesterday. 11. Thousands of new houses (build) every year. 12. This room (not use) for a long time. 13. The children (take) to the circus tomorrow.

Задание 6. Переведите предложения на английский язык.

1. Эта книга была прочитана всеми. 2. Письмо будет отправлено завтра. 3. Ее часто спрашивают? 4. На ваш вопрос ответят завтра. 5. Текст переводился вчера с двух до трех. 6. Работа только что закончена нами. 7. Эти книги будут использоваться до конца года. 8. Телеграмма уже получена? 9. О новой книге будут много говорить. 10. В нашем городе сейчас строится много новых зданий. 11. Ключи были утеряны вчера. 12. Мальчика возьмут в кино. 13. Вам сказали об этом? Задание

Задание 7. Измените предложения по образцу, используя страдательный залог вместо действительного.

Н-р:   Shakespeare wrote “Romeo and Juliet”. (Шекспир написал «Ромео и Джульетту».) – “Romeo and Juliet” was written by Shakespeare. («Ромео и Джульетта» была написана Шекспиром.)

1.                 Popov invented radio in Russia. (Попов изобрел радио в России.)

2.                 Every four years people elect a new president in the USA. ( Каждые 4 года народ выбирает нового президента в США.)

3.                 The police caught a bank robber last night. (Полиция поймала грабителя банка прошлой ночью.)

4.                 Sorry, we don’t allow dogs in our safari park. (Извините, но мы не допускаем собак в наш сафари парк.)

5.                 The postman will leave my letter by the door. (Почтальон оставит мое письмо у двери.)

6.                 My mum has made a delicious cherry pie for dinner. (Мама приготовила вкусный вишневый пирог на ужин.)

7.                 George didn’t repair my clock. (Джордж не отремонтировал мои часы.)

8.                 Wait a little, my neighbor is telling an interesting story. (Подожди немного, мой сосед рассказывает интересную историю.)

9.                 My son can write some more articles about football. (Мой сын может написать еще немного статей о футболе.)

10.            You must clean your bedroom tonight. (Ты должен убраться в своей спальне сегодня вечером.)

Задание 8. Трансформируйте предложения в страдательном залоге в отрицательные и вопросительные, переведите.

1.                 Ann was bitten by a homeless dog.

2.                 The zoo is being reconstructed at the moment.

3.                 The luggage must be checked at the customs.

4.                 Souvenirs are sold everywhere.

5.                 The job will be finished at 3 o’clock.

Задание 9. Дайте полные ответы на следующие вопросы.

1.                 Are the Olympic Games held every 10 years? (Олимпийские игры проводятся каждые 10 лет?)

2.                 Is bread made from flour or potatoes? (Хлеб готовят из муки или картофеля?)

3.                 Was the Eifel Tower built in Moscow? (Эйфелева башня была построена в Москве?)

4.                 Will the final exams be taken in summer or in winter? (Выпускные экзамены будут сдаваться летом или зимой?)

5.                 When is Christmas celebrated in Europe? (Когда празднуется Рождество в Европе?)

Критерии оценки:

5 «отлично» -  Правильное выполнение заданий. Студент отлично знает и применяет на практике грамматический материал по теме Страдательный залог английского глагола(в простых, продолженных и совершенных временах). Студент владеет необходимой  лексикой по темеГосударственное устройство, правовые институты России и англоязычных стран, обладает навыками чтения и литературного перевода.

4 «хорошо» - Правильное выполнение 8 заданий из 9. Студент хорошо знает и применяет на практике грамматический материал по теме Страдательный залог английского глагола (в простых, продолженных и совершенных временах). Студент владеет необходимой  лексикой по теме Государственное устройство, правовые институты России и англоязычных стран,  обладает навыками чтения и литературного перевода. Допускаются неточности при выполнении заданий.

3 «удовлетворительно» - Допускаются неточности и незначительные ошибки в выполнении  заданий. Студент не достаточно хорошо знает и применяет на практике грамматический материал по теме Страдательный залог английского глагола (в простых, продолженных и совершенных временах). Студент не в полной мере владеет необходимой  лексикой по темеГосударственное устройство, правовые институты России и англоязычных стран, обладает навыками чтения и литературного перевода. Объем правильно выполненных заданий составляет 6 заданий.

2 «неудовлетворительно» - Отсутствует выполнение заданий. Студент не знает и не применяет на практике грамматический материал по темеСтрадательный залог английского глагола (в простых, продолженных и совершенных временах). Студент не владеет необходимой  лексикой по темеГосударственное устройство, правовые институты России и англоязычных стран, не обладает навыками чтения и литературного перевода. Объем правильно выполненных заданий составляет менее 4 заданий из 9.

Раздел 1. Тема 1.3 Навыки общественной жизни (повседневное поведение, профессиональные навыки и умения). Модальные глаголы и их эквиваленты.

Проверяемые результаты обучения: владение навыками чтения и перевода, знание модальных глаголов и их эквивалентов, умение распознавать и употреблять их в речи.

Задания (типовые) для оценки освоения раздела 1, темы 1.3 Навыки общественной жизни (повседневное поведение, профессиональные навыки и умения). Модальные глаголы и их эквиваленты.

Задание 1. Прочитайте текст, ответьте на вопросы, данные ниже.

MY WORKING DAY

Usually I get up at 7 o'clock on week-days. I do my bed and open the window. Sometimes I do my morning exercises, but not always. Then I go to the bathroom. There I wash myself and clean my teeth. Sometimes I take a shower.   

After that I go back to my bedroom, dress myself and brush my hair. In five minutes I am ready enough for breakfast. I always have a cup of tea or coffee and a sandwich.

After breakfast I take my bag, put on my coat and go to the college. I live not far from my college and it takes me only ten J minutes to walk there. I am never late for the first lesson, because I come to the college a few minutes before the bell. I leave my coat in the cloakroom and go to the classroom.

The lessons begin at eight o'clock and at half past one they are over. Sometimes after the lessons I go to the library to I take some books. I usually do my hometask at home. As a rule, I have no free time on my week-days. . Sometimes I have dinner at my college canteen. For dinner I usually have soup for the first course, meat or fish with some salad for the second. I drink milk or a cup of tea.

Twice a week I go to the swimming pool. I play volleyball in the college team and we have our training at our big gymnasium.

In the evening all the members of our family get together. We have supper together, watch TV or read books.

Reading is my hobby. I like to read detective stories or books of modern writers. At about eleven o'clock I go to bed.

Words:

usually — обычно

enough — достаточно

shower —- душ

always — всегда

only — только

never — никогда

cloakroom — раздевалка

as a rule — как правило

canteen — столовая

twice — дважды

team — команда

gymnasium — спортзал

1. When do you usually get up on your working day?

2. What do you do in the morning when you get up?

3. What do you have for breakfast?

4. How much time does it take you to get to your school?

5. Do you have dinner at home or at the school canteen?

6. What do you usually have for dinner?

7. Do you often go to the library?

8.When does your family have supper?

9. Do you go in for sports?

10. When do you usually go to bed?

Задание 2. Ознакомьтесь с содержанием текста, дайте определение жёстким и гибким профессиональным навыкам, определите какие из них важнее, аргументируйте свой ответ.

How to develop soft и hard skills

To build a successful career, you need to regularly pump flexible and rigid skills. The development of your hard skills is studying at school and university or college, online courses, master classes, workshops, internships, intensive courses and professional conferences. With soft skills, things are more complicated. This is a constant work on yourself, the search for weaknesses and the desire to develop them.

For example, to learn how to communicate, sign up for a discussion club or arrange round tables with friends discussing various topics. For the development of project thinking, it is useful to perceive all your affairs as projects — to define a goal, divide the work into stages and set a deadline. Time management, project management, and mentoring skills will be useful here if the project is a group one. It is useful to develop emotional intelligence, which is based on empathy. Try to listen to your own and others' emotions, to monitor how they affect you, your behavior and actions.

Look into yourself, think about what soft skills you would like to develop in yourself, read modern literature on this topic, watch lectures and webinars.

Which is more important, soft or hard skills

Hard skills are characterized by diplomas and certificates confirming the skills. If you don't have professional skills, you simply won't be able to work. Therefore, many people think that the possession of rigid skills is the main thing, and they are not serious about flexible ones.

Soft skills are not quantifiable. It seems that it is not so important whether you know how to plan time as your level of Chinese. This is a mistake. According to a LinkedIn study, 57% of employers value flexible skills more than professional ones.

Flexible skills are considered by many to be the competencies of the future. Yes, it won't work without hard skills. You can't become a driver without knowing how to drive a car. But of the two nuclear physicists, most likely, the one with well-developed soft skills will build a more successful career.

Soft skills are no less important than hard ones, and both are better developed already in adolescence. Grab any learning opportunities, hard and soft skills and experience, whether it's volunteering or an online application development course.

Задание 3. Переведите предложения устно. Проанализируйте употребление модальных глаголов в следующих предложениях.

1. Who can answer my question? 2. Nobody could translate this text. 3. He ought to do this task at once. 4. Must I attend this meeting? — No, you needn't. 5. You should have shown your notes to the teacher. 6. I asked him, but he wouldn't listen to me. 7. They should visit her, she is in the hospital. 8. Last summer we would often go to the country. 9. Your son can do this work himself. 10. Would you tell me the way to the station? 11. Your friend might have informed us. 12. May I leave for a while? — Yes, you may. 13. She should be more attentive at the lessons. 14. You needn't come so early.

Задание 4. Переведите предложения на русский язык письменно.

1. We have to stay at home. 2. He was allowed to take this book. 3. Who is able to do this work? 4. He had to leave for Moscow earlier. 5. We are to take exams in June. 6. Am I allowed to visit you? 7. They were able to do this work in time. 8. I shall be able to pass my examinations. 9. She will be allowed to watch TV. 10.1 have to come in time. 11. The train is to come soon. 12. Are you able to drive a car?

Задание 5. Замените модальные глаголы соответствующими эквивалентами.

1. Не couldn't explain anything. 2. You must not stay here. 3. Can you swim? 4. You may take these books. 5. They can run quickly. 6. She might work in our room. 7. Who can read this text? 8. They must go there tomorrow. 9. May I go to the cinema? 10. We must meet at 7 o'clock.

Задание 6. Подставьте необходимые модальные глаголы.

1.                 I ... not go to the theatre with them last night, I... revise the grammar rules and the words for the test. 2. My friend lives a long way from his office and... get up early. 3. All of us... be in time for classes. 4. When my friend has his English, he... stay at the office after work. He (not)... stay at the office on Tuesday, Thursday and Saturday and... get home early. 5. ... you... work hard to do well in your English? 6. «... we discuss this question now? «No, we... . We... do it tomorrow afternoon. 7. I'm glad you... come. 8. «... you... come and I have dinner with us tomorrow? «I'd love to”. 9. «Please send them this article. «Oh, ... I do it now?»

Задание 7. Переведите на английский язык, используя модальные глаголы.

1. Мы обязательно должны писать диктант сегодня? — Да, завтра мы будем учить новые слова. 2. Вчера мне пришлось ответить на все эти письма. 3. Виктора тоже пригласить на обед? — Да, сделайте это, пожалуйста. 4. Вам пришлось остаться дома, потому что была плохая погода? 5. Вы обязательно должны прийти и посмотреть нашу новую квартиру.— С удовольствием. 6. Я рад, что мне не пришлось заканчивать эту работу вчера. 7. Я не люблю поздно ложиться спать, но иногда мне приходится. 8. Можно мне пойти погулять сейчас? — Нет, нельзя. Ты должен скоро ложиться спать. 9. Вам следует навестить вашего друга. Он вчера не пришел на урок. 10. Почему ты не пришла? — Я не могла, я должна была помочь маме по дому. 11. Вам не нужно идти в библиотеку, у нас много книг дома, и вы можете взять любую, какую хотите.

Задание 8. Заполните пропуски в предложениях соответствующими модальными глаголами (must, should, would, ought to, needn't, can, could, may, might).

1.They... not do this work themselves 2. You... take my dictionary. 3. You don't look well, you... consult the doctor. 4. Why... I give you my money? 5. She... not speak any foreign language. 6. He... to help them, they need his help. 7. ... you tell me the time? 8. ... I go with you? No, you.... 9. Your daughter... have told about it. 10. In winter we... often skate. 11. You... not miss your classes. 12. ... you play the piano before?

Задание 9. а) Подчеркните правильный вариант употребления модального глагола.

1.                 You must / should / shouldn’t be 18 before you can drive in Spain.

2.                 You don’t have to / mustn’t / shouldn’t go to bed so late. It's not good for you.

3.                 You don’t have to / mustn’t / shouldn’t wear a school uniform in most Spanish state schools.

4.                 You must / mustn’t / needn’t come. I can do it without you.

5.                 You don’t have to /must / mustn’t copy during exams.

6.                 You don’t have to /mustn’t / shouldn’t be very tall to play football.

7.                 You must /mustn’t / needn’t be a good writer to win the Pulitzer Prize.

Задание 9. б) Дополните предложения подходящим модальным глаголом, из тех, что даны курсивом, переведите.

1.                 You couldn’t / mustn’t / shouldn’t eat so many hamburgers. They're not good for you.

2.                 You can’t have / don’t have to /mustn’t study at the weekends, except when you have exams.

3.                 You may not / might not /needn’tworry. Everything will be OK.

4.                 You don’t have to /might not/mustn’t use your mobile phone in class.

5.                 Diana looks happy. She can /can have /must have heard some good news.

6.                 can’t /may not /might not have left my mobile phone at school on Friday afternoon – I had it on Friday night.

7.                 It can /could / couldn’t rain tomorrow.

Критерии оценки:

5 «отлично» -  Правильное выполнение заданий. Студент отлично знает и применяет на практике грамматический материал по теме Модальные глаголы и их эквиваленты. Студент владеет необходимой  лексикой по теме Навыки общественной жизни (повседневное поведение, профессиональные навыки и умения), обладает навыками чтения и литературного перевода.

4 «хорошо» - Правильное выполнение 8 заданий из 9. Студент хорошо знает и применяет на практике грамматический материал по теме Модальные глаголы и их эквиваленты. Студент владеет необходимой  лексикой по теме Навыки общественной жизни (повседневное поведение, профессиональные навыки и умения), обладает навыками чтения и литературного перевода. Допускаются неточности при выполнении заданий.

3 «удовлетворительно» - Допускаются неточности и незначительные ошибки в выполнении  заданий. Студент не достаточно хорошо знает и применяет на практике грамматический материал по теме Модальные глаголы и их эквиваленты. Студент не в полной мере владеет необходимой  лексикой по теме Навыки общественной жизни (повседневное поведение, профессиональные навыки и умения), обладает навыками чтения и литературного перевода. Объем правильно выполненных заданий составляет 6 заданий.

2 «неудовлетворительно» - Отсутствует выполнение заданий. Студент не знает и не применяет на практике грамматический материал по теме Модальные глаголы и их эквиваленты. Студент не владеет необходимой  лексикой по теме Навыки общественной жизни (повседневное поведение, профессиональные навыки и умения), не обладает навыками чтения и литературного перевода. Объем правильно выполненных заданий составляет менее 4 заданий из 9.

Раздел 1. Тема 1.4 Здоровый образ жизни. Сложноподчинённые предложения с союзами for, as, till, until, (as) though.

Проверяемые результаты обучения: владение навыками чтения и перевода, знание особенностей сложноподчинённых предложений с союзами for, as, till, until, (as) though, умение распознавать и употреблять их в речи.

Задания (типовые) для оценки освоения раздела 1, темы 1.4 Здоровый образ жизни.Сложноподчинённые предложения с союзами for, as, till, until, (as) though.

Задание 1. Прочитайте и запишите слова по теме Здоровый образ жизни.

1.     healthy way of life = healthy living — здоровый образ жизни

2.     unhealthy way of life = unhealthy living — нездоровый образ жизни

3.     bad/ unhealthyhabit — вреднаяпривычка

4.     take care of your health — заботиться о своем здоровье

5.     get into a habit of — завестипривычку

6.     get rid of a bad habit — избавиться от вредной привычки

7.     make it a rule — поставить за правило

8.     prefer organic food — предпочитать натуральную еду

9.     food with additives/ junk food/ fast food — еда с добавками, фастфуд

10. food rich in calories = fatty food — калорийная пища

11. influence our health — влиять на здоровье

12. improve health — улучшить здоровье

13. ruin health — навредить здоровью

14. do harm — причинять вред

15. skip breakfast — пропускать завтрак

16. be overweight — иметь избыточный вес

17. lose weight — похудеть

18. put on weight  — поправиться

19. keep to a diet / be on diet / follow a diet — бытьнадиете

20. have little physical activity — малодвигаться

21. take regular exercises — регулярно заниматься упражнениями

22. live a regular life — вести правильный образ жизни

23. a lateriser — тот, кто поздно встает

24. an earlyriser — тот, кто рано встает

25. be as fit as a fiddle — быть в добром здравии и прекрасном настроении

Задание 2. Прочитайте текст, ответьте на вопросы на понимание содержания.

«Our Health»

Our health depends on many things: our physical activity, the food we eat and our good and bad habits. Although a lot of people are interested in staying healthy, not many people do very much about it. Modern way of life when people have little physical activity, use cars instead of walking, watch television and work on computers for many hours is quite dangerous for their health. People’s  health also influences their mood.

There are many opportunities to stay healthy and be fit and one of them is going in for sports. But you needn’t be a professional sportsman. Just simple regular exercises give you energy and help you feel and look better.

Exercises that involve repeated movements such as are walking, jogging or swimming are the best. Bending and stretching which are practiced in aerobics or yoga make your body flexible and light. The cheapest and most popular sport is jogging.  If you don’t have time for it, make small changes like using stairs instead of the lift or walking or cycling instead of taking the bus and it can help you to improve your health and make you a more active person.

Also it is very important to get rid of bad habits. The worst ones are smoking and drinking alcohol. Smoking doesn’t only causes heart and lung problems but also makes your teeth yellow and skin unhealthy.

Food we eat also influences our health. A lot of people like drinking Coca-Cola and coffee and enjoy pizza and hamburgers. But what is tasty is not healthy. You should avoid eating in fast food restaurants and make it a rule to cook meals at home using organic food as much as possible.

Only a healthy man can enjoy his life, work well and be happy. There are proverbs “Health is better than wealth” and “Early to bed and early to rise makes a man healthy, wealthy and wise”.

1.Why do people care about healthy way of life nowadays?
2. How does our health depend on our lifestyle?
3. What can people do to stay healthy? What do you personally do?
4. Is sport a hobby or a part of your everyday life?
5. Is sport popular in your family? Do your parents do sports regularly?
6. Why is it important to exercise every day?
7. What bad habits do you know? Why
are hey dangerous?

Задание 3. Прочитайте текст, ответьте на вопросы к тексту, составьте план и пересказ текста.

SPORTS

Sport is very popular among people in all the countries of the world. Sport makes people healthy, keeps them fit, more organized and better disciplined. It unites people of different classes and nationalities. Many people do sports on their personal initiative. They go in for skiing, skating, table tennis, swimming, volleyball, football, basketball, body-building etc. All necessary facilities are provided for them: stadiums, sport swimming-pools, skating-rinks, football fields. But, of course, one have to pay for these services.

Sport is paid much attention to in our schools and colleges. Physical training is a compulsory subject. Different sports and games are popular with my classmates.

All my friends go in for different kinds of sport, such as water sport s (that is swimming, sailing, rowing), gymnastics, horse-racing, wrestling, fencing, weightlifting, boxing, football, basketball, volleyball etc. Physical training lessons at our college are held out -of-doors in summer. When it is cold outside the lessons are held indoors in our college gymnasium.

Professional sport is also paid much attention to in our country. In the city where I live, there are different sport societies, clubs and sport schools.

Practically all kinds of sports are popular in our country, but gymnastics and tennis enjoy the greatest popularity.

The most popular kinds of sports in the United States are baseball, basketball and American football.

In England popular kinds of sports are golf and rugby. Englishmen like football too. It is their national kind of sports.

Words:

all over the world — во всем мире

to be fond of — любить

healthy — здоровый

to be fit — быть в форме

personal initiative — личная инициатива

skiing — лыжный спорт

skating — коньки

body-building — культуризм

facilities — помещения, оборудование

swimming-pool — плавательный бассейн

skating-rink — каток

attention — внимание

compulsory — обязательный

sailing — парусный спорт

rowing — гребля

wrestling — борьба

fencing — фехтование

weightlifting — штанга

out-of-doors — навоздухе

indoors — в помещении

sport societies — спортивные общества

Questions:

 1. Why do people all over the world are fond of sports and games?

 2. What are summer sports and what are winter sports?

 3. What kind of sports are popular with your friends?

 4. Do you have to pay for sports facilities, such as stadiums, swimming pools and tennis courts?

 5. What can you say about physical training lessons at your college?

 6. What kinds of sport are most popular in our country?

 7. What are the sports organisations in our country?

 8. What are the most popular kinds of sport in America and England?

 9. What other American or English kinds sports do you know?

 10. Are American football and baseball popular in Russia? Whyandwhynot?

Задание 4.Заполните таблицу следующими видами спорта. Решите,

какие виды спорта употребляются со словами PLAY, GO и DO.

Football, boxing, golf, karate, skiing, rugby, table tennis, volleyball, basketball, gymnastics, yoga, swimming, hockey, running, judo, soccer, bowling, dancing, cycling, chess

Задание 5. Угадайте, какой вид спорта описывается в каждом предложении.

1.                 Each team has eleven players. The players of the two teams wear clothes of different colors. Only the goalkeeper can touch the ball with the hands.

2.                 Each team has up to eleven players, but only seven of them can play at the same time. The players have caps on. They mustn’t splash water into the face of opponent.

3.                 Each team has six players on the court. The player can hit the ball with the hand. She/he has to release the ball before hitting it. The players are not allowed to touch the net.

4.                 It is a team sport. Each team has up to ten players, but only five of them can play at the same time. The players must try for a goal within 30 seconds of possessing the ball.

5.                 Each team can have up to seventeen players, but only six can play at the same time. Players wear skates and helmets.

Задание 6.  Дополните текст подходящими по смыслу словами из рамки по теме спорт.

pools / courts / stadium / rink / athletes / jumping / track / pitches / rings 

Nick James is very happy because there is a big sports center near his home. There are football (1) _______, tennis and basketball (2) ________, swimming (3) _______, a sports hall with two boxing (4) ______ and even a skating (5) _______. There is also a separate athletics (6) _______, where 30 000 spectators can watch track events on the (7) _____ and field events, such as (8) ________and throwing, in the grass center. The (9) _______ get ready in modern changing rooms. A huge scoreboard shows the results. Nick often attends sports events. He also goes swimming three times a week and hopes to succeed in this sport.

Задание 7. Выберите в скобках верный вариант союза времени. Переведите сложноподчиненные предложения.

1.                 Please, call me ... (while/as soon as/till) you arrive at the station.

2.                 ... (Until/After/Before) going to sleep, she likes reading a book for an hour or so.

3.                 Don’t sit down ... (until/after/while) we ask you.

4.                 The house became empty ... (after/before/until) they left.

5.                 I won’t start my journey ... (when/while/until) he pays all my expenses.

6.                 ... (as soon as/while/before) he was making a fire, it started raining.

7.                 ... (while/till/as soon as) we get married, we’ll move to another place.

8.                 Turn off the light ... (before/when/while) the lesson is over.

9.                 You should read this paper ... (before/until/while) you express your opinion.

10.            We won’t start our dinner ... (after/until/as soon as) Richard arrives.

Задание 8. Переведите на английский язык следующие сложноподчиненные предложения.

1) Я не смогу ответить на ваш вопрос то тех пор, пока не получу от них письма.

2) Он упал, когда выходил из автобуса.

3) Они сидели в парке до тех пор, пока не стало совсем темно.

4) Когда я шел сюда, я встретил старого друга.

Задание 9. Соедините два простых предложения в одно сложноподчиненное, используя предложенный союз в скобках.

Например:  Don’t call your uncle. First I’ll tell you. (till) – Don’t call your uncle till I tell you. (Не звони своему дяде, пока я не скажу тебе.)

1.                 Tom will come in a minute. Then I will go to the chemist’s. (as soon as)

2.                 I will come home soon. I want to have a foam bath. (when)

3.                 We are moving to Miami. Then we’ll invite you to our house. (after)

4.                 You will need my car. But first let me know. (before)

5.                 Alice will be picking up the berries. Sam will be cutting the grass. (while)

6.                 She will have finished her homework in half an hour. Then she will go for a walk. (after)

7.                 Dad will have repaired the van by Saturday. Then we’ll travel to the lakes. (as soon as)

8.                 You must clean your teeth. Then you may go to bed. (until)

9.                 I will get my salary next week. Then I will buy a new smart phone. (when)

10.            First wait for the green light on. Then you can cross the street. (before)

Критерии оценки:

5 «отлично» -  Правильное выполнение заданий. Студент отлично знает и применяет на практике грамматический материал по теме Сложноподчинённые предложения с союзами for, as, till, until, (as) though. Студент владеет необходимой  лексикой по теме Здоровый образ жизни, обладает навыками чтения и литературного перевода.

4 «хорошо» - Правильное выполнение 8 заданий из 9. Студент хорошо знает и применяет на практике грамматический материал по темеСложноподчинённые предложения с союзами for, as, till, until, (as) though. Студент владеет необходимой  лексикой по теме Здоровый образ жизни, обладает навыками чтения и литературного перевода. Допускаются неточности при выполнении заданий.

3 «удовлетворительно» - Допускаются неточности и незначительные ошибки в выполнении  заданий. Студент не достаточно хорошо знает и применяет на практике грамматический материал по теме Сложноподчинённые предложения с союзами for, as, till, until, (as) though. Студент не в полной мере владеет необходимой  лексикой по теме Здоровый образ жизни, обладает навыками чтения и литературного перевода. Объем правильно выполненных заданий составляет 6 заданий.

2 «неудовлетворительно» - Отсутствует выполнение заданий. Студент не знает и не применяет на практике грамматический материал по теме Сложноподчинённые предложения с союзами for, as, till, until, (as) though. Студент не владеет необходимой  лексикой по теме Здоровый образ жизни, не обладает навыками чтения и литературного перевода. Объем правильно выполненных заданий составляет менее 4 заданий из 9.

Раздел 1. Тема 1.5 Профессии, карьера. Сложносочинённые предложения. Сложноподчинённые предложения. Типы придаточных  предложений.

Проверяемые результаты обучения: владение навыками чтения и перевода, знание особенностей сложносочиненных и сложноподчиненных предложений, типов придаточных предложений, умение распознавать и употреблять их в речи.

Задания (типовые) для оценки освоения раздела 1, темы 1.5 Профессии, карьера. Сложносочинённые предложения. Сложноподчинённые предложения. Типы придаточных  предложений.

Задание 1. Перепишите слова с переводом.

1.       curriculum vitae (CV)                   биография, резюме

2.       written application                       письменное заявление

3.       opening position                          вакансия

4.       to choose an occupation               выбрать работу по

to one's liking                     желанию

5.       experimental period                     испытательный срок

6.       to work shift-work                       работать по сменам

7.       to have flexible schedule              иметь гибкий график

8.       to work part/full time                    работать неполный/полный рабочий день

9.       to work nine to five                      работать с девяти до пяти

10.   to get off                                       увольняться

11.   be on sick leave                            быть на больничном

12.   to run business                             заниматься бизнесом

13.   to do trade                                   вести торговлю

14.   entrepreneur                                 предприниматель

15.   owner                                          владелец

16.   officer                                          чиновник, должностное лицо

17.   employment office                        агентство по трудоустройству

18.   work experience                          стаж

19.   employment book                        трудовая книжка

20.   position                                       должность

21.   supervisor                                    начальник

22.   subordinate                                  подчиненный

23.   salary                                          зарплата служащего

24.   wage                                            зарплата рабочего

25.   fee                                                гонорар творческого работника

26.   rate of pay                                   ставка оплаты

27.   fair salary                                    достойная зарплата

28.   legal contract                               трудовое соглашение

29.   incometax                                    налог

30.   contractual obligation                  договорные обязательства

31.   to undertake a responsible post     занимать ответственный пост

32.   employer/employee                      служащий/работодатель

33.   jobprospects                                 перспективы для работы

34.   opportunities for rapid                  возможности

35.    advancement                               быстрого роста

36.   to be accustomed to work             привыкнуть

under pressure                             напряженно работать

37.   to contact with people easily         легко налаживать контакт с людьми

38.   to have good reference                  иметь хорошие

отзывы

39.   supportive environment                хорошая рабочая

обстановка

40.   main strength                               основные достоинства

41.   promising                                    перспективный

42.   creative                                        творческий

43.   reliable                                         надежный

44.   skilful                                          опытный

Задание 2. Прочитайте текст, составьте план и краткое изложение содержания текста.

Planning a career

Having a job and having a career are two very different things. A job is something you do to make money. You may enjoy the job, work hard at it and do well, but you are primarily doing it for the money to satisfy your other interests outside of the work environment. A career is something that integrates your desires and interests so that it gives you satisfaction above and beyond the money you make. A profession is a type of job that needs a high level of education. To have a career means commitment and development but first of all planning.

This process can begin at any age. For some people it starts when they are small children and visit mom or dad at their place of work. For others it can come later through the inspiration of a teacher or exposure to a wider range of fields.

It is up to each individual to decide whether a job or career is best for them. People may share the same talent and interest but other aspects of their personality will dictate which direction to go with that interest. For example, one guitar player may decide to plan a career as a professional musician. Another may decide that the financial insecurity is too much for him, get a regular job satisfy his musical interests in his free time.

Whether you decide to get a job or plan a career, the job market today is quite different from that of your parents. In the Soviet system young people were guaranteed a job upon graduation. Now, there are no guarantees after university, institute or school.

The young person in today's Russia faces a very competitive job market.

What do the new dynamics of the Russian job market mean for young people? First, if they have decided they want a career, they must start early in their academic life to plan and take steps to develop their professional careers. Second, in addition to a suitable background for a desired career, creativity, self-promotion and preparation are absolutely vital for any sort of success in the job search. Last, students must develop confidence in themselves and recognize the power that each of them has to take control of their future and shape it in a way that is best for them.

Задание 3. Прочитайте следующие утверждения, определите правдивы они или ложны. Read the text and say if you agree or disagree with the following statements: true or false.

1.     Having a job and having a career are two very different things.

2.     Planning a career can begin at any age.

3.     The choice of a career doesn't only depend on a person's talents and interests.

4.     To face a competitive job market is to have no guarantees for getting a job.

5.     To take control of the future and to be well-prepared for the challenges of the job market one should take several very important steps.

Задание 4. Ответьте письменно на следующие вопросы.

1.     What does your father do for a living?

2.   Is there a lot of stress connected with his work?

3.   Do you think that present career is the right one for him?

4.   What is your mother's occupation?

5.   Have your parents ever been unemployed?

6.   What is your family income?

7.   Is career an important part in your life?

8.   What is your idea of an ideal job?

9.   What are some good jobs to have and why?

10. What are the worst jobs and why?

11. What job would you like to get after you graduate from the University?

12. You have won or inherited a lot of money. Would you continue working?

13. Would you agree to get married and not to work?

14. At what age can you get a part-time job in your country?

15. What are the most popular jobs for young men and women?

16. Would you like your work to be indoors or outdoors?

17. Would you like to have your own business? Why?

18. Do you prefer to have a job for which no further training is required or further training is necessary?

19. Would you like to work for a big organization?

20. Would you like a job that involved making things with your hands?

21. What does you future profession demand from you?

22. What are the main advantages and disadvantages of your future profession?

Задание 5. Составьте монологическое высказывание о своей будущей профессии по ответам на следующие вопросы. Speak on your future profession in as many details as possible, answering the following questions:

1. Where do you study?

2. What faculty and speciality do you study at?

3. What general and special subjects do you have?

4. Why did you choose this profession?

5. Do you have practice at enterprises of our town?

6. Is your future profession useful? Why?

Задание 6. Заполните пропуски в предложениях требуемыми по смыслу словами. Complete the sentences with the words:

Occupation, career, creative, architect, vet, responsible, noble, prestigious, librarian, fireman.

1. I think teaching is a ……….. profession.

2. We have always thought that any job in the hospital is………

3. Careers of computers programmers are very………. nowadays.

4. My cousin wants to be an………

5. Will you write your…….. on this form?

6. A……. is a person who works in the library to help people to choose books to read.

7. I am sure that the profession of a………… is rather dangerous.

8. His ………. As a driver came to the end after a road accident.

9. I am going to be a……. because I like animals and birds.

10.            The profession of a photographer is very…………

Задание 7. Вставьте союзы там, где необходимо в данных сложносочиненных предложениях.

1.  Nowadays he was busy  he saw few of his old friends.

2.  He  spoke better, … also he spoke more correctly.

3.  I have not much news to convey  there are some things to add.

4.  Honey is sweet, … the bee stings.

5.  … you must improve your work, … I shall dismiss you.

6.  You can take a bus, … we can walk together.

7.  The signal was given … the steamer moved slowly from the dock.

8.  We move to a new flat … that’s settled.

9.  The rain stopped…the sky cleared: we could continue our journey.

10.  He speaks English, … his sister speaks German.

11.  I like the sports channel, … my wife prefers movies.

12.  We overslept … the train left without us.

13.  I saw hundreds of people on the streets … many of them looked familiar.

14.  The weather is glorious … we are sure to win.

Задание 8. Переведите сложноподчиненные предложения на русский язык, определите их тип.

1.  As long as you are working here, we’ll have a rest.

2.  His words that he wasn’t coming upset me.

3.  They hurry lest they shouldn’t be late.

4.  Wherever it was possible, the travelers camped for the night.

5.  We came half an hour earlier so that you could speak to him.

6.  The TV box which we bought yesterday is very good.

7.  I don’t know what you are talking about.

8.  She is smiling because she has remembered something funny.

9.  The man who called didn’t say his name.

10.  As soon as he came, they started the work.

11.  Write down all the new words lest you should forget them.

12.  This girl says she can speak three languages.

13.  The book you are discussing is unfamiliar to me.

14.  They waited for him where he usually walked.

15.  As it was warm, he went for a walk.

16.  He speaks slowly in order that we may understand everything.

Задание 9. Переведите следующие сложноподчиненные предложения на английский язык.

1.  Мои друзья обещают мне, что помогут мне.

2.  Джон объяснил, почему он опоздал.

3.  Так как было холодно, мы не пошли гулять.

4.  Я ушел, так как там никого не было.

5.  Он сказал, что собирается уехать из нашего города.

Критерии оценки:

5 «отлично» -  Правильное выполнение заданий. Студент отлично знает и применяет на практике грамматический материал по теме Сложносочинённые предложения. Сложноподчинённые предложения. Типы придаточных  предложений. Студент владеет необходимой  лексикой по теме Профессии, карьера, обладает навыками чтения и литературного перевода.

4 «хорошо» - Правильное выполнение 8 заданий из 9. Студент хорошо знает и применяет на практике грамматический материал по теме Сложносочинённые предложения. Сложноподчинённые предложения. Типы придаточных  предложений. Студент владеет необходимой  лексикой по теме Профессии, карьера, обладает навыками чтения и литературного перевода. Допускаются неточности при выполнении заданий.

3 «удовлетворительно» - Допускаются неточности и незначительные ошибки в выполнении  заданий. Студент не достаточно хорошо знает и применяет на практике грамматический материал по теме Сложносочинённые предложения. Сложноподчинённые предложения. Типы придаточных  предложений. Студент не в полной мере владеет необходимой  лексикой по теме Профессии, карьера,  обладает навыками чтения и литературного перевода. Объем правильно выполненных заданий составляет 6 заданий.

2 «неудовлетворительно» - Отсутствует выполнение заданий. Студент не знает и не применяет на практике грамматический материал по теме Сложносочинённые предложения. Сложноподчинённые предложения. Типы придаточных  предложений. Студент не владеет необходимой  лексикой по теме Сложносочинённые предложения. Сложноподчинённые предложения. Типы придаточных  предложений, не обладает навыками чтения и литературного перевода. Объем правильно выполненных заданий составляет менее 4 заданий из 9.

 

Раздел 1. Тема 1.6 Отдых, каникулы, отпуск. Туризм. Вспомогательные глаголы to be, to have, to do в английском языке.

Проверяемые результаты обучения: владение навыками чтения и перевода, знание форм и случаев употребления вспомогательных глаголов to be, to have, to do, умение употреблять их в речи.

Задания (типовые) для оценки освоения раздела 1, темы 1.6 Отдых, каникулы, отпуск. Туризм. Вспомогательные глаголы to be, to have, to do в английском языке.

Задание 1.Ознакомьтесь с текстом и словами к тексту.

TRAVELLING

Travelling became a part of our life. Thousands of people travel every day either on business or for pleasure. They travel by road, by train, by air or by sea. Of course, travelling by air is the fastest and the most convenient way, but it is the most expensive, too.

Travelling by train is slower than travelling by plane, but it is less expensive. You can see many interesting places of the country through the window. Modern trains have more comfortable seats. There are also sleeping cars and dining cars that make even the longest journey more pleasant. Speed, comfort and safety are the main advantages of trains and planes. That is why many people prefer them to all other kinds of travelling.

Travelling by sea is popular mostly for pleasure trips. Tourists can make voyages on large ships to foreign countries. The trips on the Volga, the Don and the Black Sea are very popular today.

As for me I prefer travelling by car. I think it's more convenient because you don't buy tickets, you can stop any place and spend as much time as you like at any place.

Words:

either ... or — или ... или

train — поезд

fast — быстрый

convenient — удобный

way — путь, способ

slow — медленный

advantages — преимущества

plane (airplane) — самолет

expensive — дорогой (остоимости)

sleeping car — спальный вагон

dining car — вагон-ресторан

journey — путешествие, поездка

kind — вид, род, сорт

pleasant — приятный

mostly — главным образом

voyage — путешествие (поводе)

trip — путешествие

to prefer — предпочитать

Задание 2. Ответьте на вопросы к тексту.

1) Do you think modern life is impossible without travelling?

2) Why do people travel?

3) What are the means of travelling?

4) How do you prefer to travel and why?

5) What is the fastest kind of travelling?

Задание 3. Прочитайте и устно переведите текст

Tourism and types of tourism

Tourism is defined as the act of travel with the intentions of recreational pleasure. The World Tourism Organization defines a tourist is someone who travels at least 50 miles or 80 kilometers away from their home, for the purpose of entertainment and pleasure. The terms tourist and tourism were first recognized in 1937 by the League of Nations, whose definition involved a person who traveled abroad for more than 24 hours. Tourism is the act of paying money to go from one place to another to see different and unique sights, has been a fact of civilized life since approximately the 12th century.

Types of tourism

There are different types of tourism that can be enjoyed. Some are listed below:

· Extreme tourism or shock tourism is a type of niche tourism involving travel to dangerous places (mountains, jungles, deserts, caves, etc.) or participation in dangerous events.

· Cultural tourism involves visiting historical or interesting cities, such as Paris, Shanghai, Rome or Warsaw. This is when tourists engage in cultural experiences, like visiting an art museum, theater or opera.

· Ecotourism involves traveling that does not pose a threat to the environment, such as safariing in Kenya. Ecotourism integrates tourism with ecology, offering wide varieties of landscapes and activities.

· Educational tourism developed because of the growing popularity of teaching and learning of knowledge, and enhancing technical competency outside the classroom environment.

Задание 4. Дайте краткую характеристику экстремального, культурного, экологического и образовательного туризма.

Extreme tourism

Extreme tourism or shock tourism is a type of niche tourism involving travel to dangerous places (mountains, jungles, deserts, caves, etc.) or participation in dangerous events. Extreme tourism overlaps with extreme sport. The two share the main attraction, "adrenaline rush" caused by an element of risk, and differing mostly in the degree of engagement and professionalism. Extreme tourism is a growing business in the countries of the former Soviet Union (Russia, Ukraine, Azerbaijan, etc.) and in South American countries like Peru, Chile and Argentina.

The mountainous and rugged terrain of Northern Pakistan has also developed into a popular extreme tourism location. While traditional tourism requires significant investments in hotels, roads, etc., extreme tourism requires much less to jump-start a business. In addition to traditional travel-based tourism destinations, various exotic attractions are suggested, such ice diving in the White Sea, or travelling across the Chernobyl zone.

Demand for extreme tourism in Russia is greatly increased. Tourist’s firms actively offer rafting, traveling on horse back, by bicycle and motorcycle. More and more people are attracted by rafting, diving, pleasure flight on balloon and many others. Russians are testing their nerves more and more often nowadays. Extreme tourism is becoming very popular in the country.

Extreme tourists Fyodor Konyukhov, Dmitri and Matvei Shparo have become symbols of Russian fearlessness, and their names are skillfully used as brand names by manufacturers of tourist equipment.

Diving is very popular in the whole world. It is underwater diving with special apparatuses, providing a swimmer with breathing. Diving is both a sport and entertainment. Recently, diving has become one of the trendiest varieties of extreme tourism in Russia. As many as 15,000 Russians practice diving. They enjoy underwater scenery not only in warm seas - the Black Sea, for example, but also in cold waters of the Arctic Ocean.

Cultural tourism

Cultural tourism is the subset of tourism concerned with a country or region's culture, especially its arts. It generally focuses on traditional communities who have diverse customs, unique form of art and distinct social practices, which basically distinguishes it from other forms of culture.

Cultural tourism includes tourism in urban areas, particularly historic or large cities and their cultural facilities such as museums and theatres. It can also include tourism in rural areas showcasing the traditions of indigenous cultural communities and their values and lifestyle. It is generally agreed that cultural tourists spend substantially more than standard tourists do.

This form of tourism is also becoming generally more popular throughout Europe. On the positive side are the unique cultural practices and arts that attract the curiosity of tourists and provide opportunities for tourism and economic development.

On the negative side is the issue of how to control tourism so that those same cultural amenities are not destroyed and the people do not feel violated.

Ecological tourism

According to the definition and principles of ecotourism established by The International Ecotourism Society (TIES) in 1990, ecotourism is "Responsible travel to natural areas that conserves the environment and improves the well-being of local people."

Ecotourism (also known as ecological tourism) is a form of tourism, that appeals to ecologically and socially conscious individuals. Generally speaking, ecotourism focuses on volunteering, personal growth and learning new ways to live on the planet.

It typically involves travel to destinations where flora, fauna and cultural heritage are the primary attractions. Many locations have bеcоmе popular because of the growing worldwide interest in ecology.

Ecotourism integrates tourism with ecology, offering wide varieties of landscapes and activities, including unspoiled bеасhes and coral reefs with productive marine systems, for scuba-diving enthusiasts; vast limestone caverns in highlands with trekking, mountain biking, sailing, rafting and other travel opportunities and miles and miles of empty beaches.

Responsible ecotourism includes programs that minimize the negative aspects of conventional tourism on the environment and enhance the cultural integrity of local people. Therefore, in addition to evaluating environmental and cultural factors, an integral part of ecotourism is the promotion of recycling, energy efficiency, water conservation and creation of economic opportunities for the local communities.

Number of fans of ecotourism grows everywhere every year. This kind of tourism return people, tired of cities, forces and energy during contact with the nature and observation of it. The list of the UNESCO world heritage includes 5 Russian natural objects: primeval forests of Komi, Baikal lake, volcanoes of Kamchatka, golden Altai mountains, Western Caucasus. In these regions just ecotourism can help to conservancy.

Educational tourism

Educational tourism developed because of the growing popularity of teaching and learning of knowledge, and enhancing technical competency outside the classroom environment.

In the educational tourism, the main focus of the tour or leisure activity includes visitation of another country to learn about the culture of the visited country (Student Exchange Program and Study Tour) or to work and apply their learning inside the classroom in different environment (International Practicum Training Program).

Задание 5.Ответьте на вопросы письменно.

1. Tourism is defined as the act of travel with the intentions of recreational pleasure, isn`t it? 2. What does The World Tourism Organization define? 3. Which types of tourism do you know? 4. What form of tourism is also becoming generally more popular throughout Europe? 5. What can the ecotourism do in these regions?

Задание 6. Дополните предложения, используя необходимую форму глагола to be. Complete what Brenda says about herself. Use am, is or are.

My name (1) ______ Brenda Foster. I (2)  ______ on the left in the picture. I (3) ______  ten years old and I (4) ______ in the fifth form. My birthday (5)  _____ on the first of January. I (6) ______  from Santa Monica, California, USA. I (7) ______ American. My phone number (8) ______ 235-456-789. I live at 16 Park Street. My post code (9) ______ LA 30 SM. I’ve got a sister and a brother. Their names (10) ______ Gina and Paul. Gina (11) ______ 16 years old and Paul (12) ______  only three. I’ve also got a dog. His name (13) ______ Spot. He (14) ______  on the right in the picture. My Mum (15) ______ a doctor. She works at a hospital. My Dad (16) ______ a driver. He works in Los Angeles. We (17)  ______ all   friendly in our family.

Задание 7. Дополните предложения, используя нужную форму глагола to be в прошедшем времени. Write in was / were

1.                 _________ Jenny at the party?

2.                 _________ Lumpy quiet yesterday?

3.                 _________ you in Kongo?

4.                 _________ your parents in the local gym yesterday?

5.                 _________ your friend at school yesterday?

6.                 _________ you happy  yesterday?

7.                 _______     your mum tired yesterday?

Задание 8. Заполните пропуски глаголом have got / has got, сделайте предложения вопросительными и отрицательными.

1The child  …….. a new beautiful toy.

2. They …… eight beautiful flowers.

3. Ann …… a nice black piano.

4. You …… two beds in your room.

5. The woman …… a very nice dress.

6. Tim ….. three bananas.

7. I …… seven cousins.

8. Diana and George …..  four children.

9. Jane  …….  two  uncles.

10. We …….   five English books.

Задание 9. Вставьте do/ does в вопросительные предложения.

1.                 _____ you go to school on Sundays?

2.                 _____ Molly study well?

3.                 _____ you clean your room every day?

4.                 _____ your Mum feed your dog in the morning?

5.                 _____ Ben live in Brighton?

6.                 _____ Oleg go in for football?

7.                 _____ Tom drive his car well?

8.                 _____ she get up early every day?

9.                 _____ you like rainy weather?

10.            _____ Tim like to go to the theatre?

Критерии оценки:

5 «отлично» -  Правильное выполнение заданий. Студент отлично знает и применяет на практике грамматический материал по теме Вспомогательные глаголы to be, to have, to do. Студент владеет необходимой  лексикой по темеОтдых, каникулы, отпуск. Туризм, обладает навыками чтения и литературного перевода.

4 «хорошо» - Правильное выполнение 8 заданий из 9. Студент хорошо знает и применяет на практике грамматический материал по теме Вспомогательные глаголы to be, to have, to do. Студент владеет необходимой  лексикой по теме Отдых, каникулы, отпуск. Туризм, обладает навыками чтения и литературного перевода. Допускаются неточности при выполнении заданий.

3 «удовлетворительно» - Допускаются неточности и незначительные ошибки в выполнении  заданий. Студент не достаточно хорошо знает и применяет на практике грамматический материал по теме Вспомогательные глаголы to be, to have, to do. Студент не в полной мере владеет необходимой  лексикой по теме Отдых, каникулы, отпуск. Туризм, обладает навыками чтения и литературного перевода. Объем правильно выполненных заданий составляет 6 заданий.

2 «неудовлетворительно» - Отсутствует выполнение заданий. Студент не знает и не применяет на практике грамматический материал по теме Вспомогательные глаголы to be, to have, to do. Студент не владеет необходимой  лексикой по теме Отдых, каникулы, отпуск. Туризм. Не обладает навыками чтения и литературного перевода. Объем правильно выполненных заданий составляет менее 4 заданий из 9.

Раздел 1. Тема 1.7Проблемы молодежи. Виды развлечений современной молодёжи. Сослагательное и повелительное наклонение в английском языке.

Проверяемые результаты обучения: владение навыками чтения и перевода, знание особенностей образования сослагательного и повелительного наклонения, умение распознавать и употреблять их в речи.

Задания (типовые) для оценки освоения раздела 1, темы 1.7 Молодежные течения. Виды развлечений современной молодёжи. Сослагательное и повелительное наклонение в английском языке.

Задание 1. Ознакомьтесь с лексикой по теме Проблемы молодежи «Problems of the Young».

a rule — правило

strict / kind — строгий / добрый

fair / unfair — справедливый / несправедливый

rude / polite — грубый / строгий

agressive / violent — агрессивный

indulgent — потворствующий

to need in — нуждаться в

necessary for — необходимый

be responsible for — отвечать за что-то

private life — личная жизнь

own business — личное дело

attitude to — отношение к

to interfere — вмешиваться

a permission — разрешение

ask for permission — спрашивать разрешения

to allow — разрешать

I am allowed — Мне разрешают

To make smb do smth — заставлять (после глагола «make»  в значении «заставлять» частица to не ставится)

to influence smb — влиять

to depend on зависеть от

to face problems — сталкиваться с проблемами

to cope with problems (= overcome problems)– справляться с проблемами (преодолевать)

to quarrel with  — ссоритьсяс

to argue with — споритьс

to agree/ disagree with — (не) соглашатьсяс

to contradict — возражать

to answer back — огрызаться

to defend one’s opinion — отстаивать свое мнение

to be on good terms with (= get on well with) — быть в хороших отношениях

to punish (punishment) — наказывать (наказание)

reason / cause — причина

to cause — вызывать

to lead to — вести к

to understand — понимать

to misunderstand — понимать неправильно

misunderstanding — непонимание

to behave — вести себя

behavior — поведение

to follow parents’ advice — следовать совету родителей

to consult parents/ friends — советоваться с родителями

to insist on doing– настаивать на

to come across the same problem — встречаться с той же проблемой

to treat as a child — обращаться как с ребенком

to make fun of — высмеивать

to respect — уважать

to rely on положиться на

have much in common — иметь много общего

to trust — доверять

to betray — предавать

to envy — завидовать

Задание 2. Прочитайте текст, письменно переведите.

The Problem of Misunderstanding with Parents or Teachers

Problems with parents. Personal problems can look silly and unimportant to the eyes of grown-ups, who have already passed this period. These problems are caused by generation gap. Every generation is unique in its experience. It has its own ideals and a system of values.

Adults always complain that the young are not always what they were. These words are repeated from generation to generation. That’s correct. In fact today the young are better educated. They grow up more quickly. They have more freedom.

At present the young do not blindly accept the ideals of their parents.  They don’t believe that they are right only because they are older. But the adults don’t want their values to be questioned. All these differences generate a generation gap when the young and adults don’t understand one another.

As a rule, the adults dissatisfied with their own life, teach the young how to live. Unfortunately they apply old standards to the new way of life.  But the young people don’t want to live in the past. They have their own ideas. They want to make their own mistakes rather than to listen to the warnings of the adults and repeat the mistakes of the older generation. They want to overcome their own difficulties. But grown-ups try to keep teenagers away from all the “dangers”, which makes up life.

Problems with teachers. The same misunderstanding we face sometimes from the side of the teachers. They think that youth is given for studying.  However, school takes all teenagers’ time but the person who only studies is dull.

Another problem is bad marks. Bad marks don’t encourage weak students, they simply can’t do better. Also bad marks can cause problems with parents as they don’t want to understand that their child does his (her) best.

Задание 3.Ответьте на вопросы.Answer the questions.

1.                 Do you often quarrel with your parents?

2.                 Do you have much in common with your parents?

3.                 Do you always understand your parents? Do they understand you? What are the reasons of misunderstanding?

4.                 Do your parents help you to solve your problems?


Задание 4. Прочитайте тексты о проблемах молодежи (проблемы в общении/ проблема зависимостей/ проблема профессионального самоопределения / буллинг (травля)). Прокомментируйте 3 проблемы по выбору на английском языке, выражая свою позицию.

The Problem of Communication / Problem with Friends

Communication has always been an important part of young people’s life. It is very difficult to be on good terms (get on) with everybody as all people are different. Sometimes teenagers don’t feel comfortable in a group because they are too shy or not very strong. But every human needs to socialize and it is the reason why they prefer different ways of communication.

Today teenagers can choose between traditional and new ways of communication. Traditionally they meet after classes with their school friends, make parties, go to the cinema or disco clubs to have fun, relax and make new friends.

But if the young people do not like noisy clubs and other places of entertainment, they can find friends without leaving their homes – the Internet gives them such an opportunity. They can sit for twenty hours chatting with their friends.

The Problem of Addiction / Health Problems

Every type of addiction is dangerous for a personality. For an addiction we mean the unability to stop doing actions that are harmful for health.

Let us take for example, computer games. Nowadays many young people are keen on computer games. Such young people don’t eat, sleep, work or learn properly. It’s a great problem and parents don’t know how to make them get interested in a real life.

The tragic effect of drug-addiction is experienced by millions of people. The use of drugs starts at school, at parties, in discotheques, etc, when youngsters are no more than 15 to 17 years old. Often they make their first “try” out of curiosity – or so as not to be “different” from their “mates”. They want to get new, unexpected sensations, they hope it will stimulate their imagination and creative abilities. But, on contrary, drugs actually undermine creativity.

As a result of regular use of drugs an active and lively personality turns into a listless person, lacking in energy and without any interests. Life becomes focused on the need to get and apply drugs. It means that the total personality becomes influenced by the regular use of drugs. The addict gradually loses his or her better moral characteristics, becomes detached from friends and family. Drugs don’t mean enjoyment, they mean dependence. And it is too late when all the victims realize it. It can take a year or six months for that “dependence” to develop – but more often, just a month or two. Sometimes a person can become addicted after just one injection.

Young people can have health problems caused by smoking or drinking beer. They do it to look older and more independent.

Problem of Choosing Future Profession / Problem of Education

The problem of education is also very serious. There are educational institutions at present but the quality of education is different. Some of are state, some are private. Some are difficult to enter, others are easy to enter. Some of them prepare specialists who can easily find a prestigious and well-paid job, others offer their graduates only poor-paid positions.

The choice of the educational institutions depends on the school you study at, your parents, the financial conditions of the family, your likes and dislikes. But everybody has to study hard because to enter a good institute they must have good score on their exam test.

It is a great problem to find a suitable institute and a profession for the rest of your life. But it is very important as making the wrong choice will influence your future life.

Bulling

Bulling is a problem which is wide-spread at schools and in neighbor hoods.

A bully is anyone who feels powerful when they hurt or pick on someone else. They do not need a reason, just a victim to tease, threaten, cause harm or abuse. If you are a victim of a bully, there are a few things you can do:

·                     Avoid the bully whenever possible. Walk or run away from the bully.

·                     Ask your friends to stick close. It is harder to pick on someone when they have friends standing up for them.

·                     Ask an adult (parent, teacher or counselor) for help.

·                     Be brave and firm. Tell the bully to stop picking on you, that you do not like it and will report the abuse if it contin­ues.

If you see someone else being bullied, don’t get in the way; you could be hurt. Find an adult to help ASAP (As Soon As Possible).

Задание 5. Пройдите тест, закончив предложения о проблемах молодежи, используя 1 из предлагаемых вариантов:

1.Being a teenager is not so...

a) important b) easy c) clever

2. Young people have to learn ... at school.

a) a little b) nothing c) a lot of

3. When parents don't understand the problems of their children it's

so-called...

a) generation gap b) problem c) the rush hour

4. Nowadays we have very serious problems for the young people such as violence, AIDS, drugs and ...

a) music b) alcohol c) sport

5. The most terrible thing is when teenager becomes a ...

a) student b) doctor c) drug addict

6. Teenagers commit crimes for some reasons. They need money, someone makes them do it or they want to…. their friends.

a) like b) impress c) watch

7. The most popular subcultures are Hacker, Rocker, Punk, Mod, Hippie, Biker and…

a) Goth b) Student c) Stranger

8. A young person who wears leather jacket, army boots and a cowboy hat just to show off. He is associated with something rebellious.

a) Hippie b) Rocker c) Mod

9.A young person who dresses in a shocking way to express his or her identity. He or she has brightly coloured hair and wears metal chains. The person is thought to rebel against the society. The music is aggressive. They reject everything.

a) Punk b) Hacker c) Hippie

Задание 6. Проинтервьюируйте друг друга по следующим вопросам о проблемах молодежи.

1.     Do you know about your friend’s problems? Do you try to help? What do you value in your friends?

2.     If you are keen on computer games, you don’t have any friends, aren’t you?

3.     Only weak people can be addicted, can’t they?

4.     Why is it important to make the right choice while choosing future profession?

5.     Why is it difficult?

6.     Is it necessary to study hard to make a successful career? Why or why not?

7.     Is there bullying in your technical school?

8.     Have you ever seen someone been bullied? What did you do?

9.     What do you think is the best way to stop a bully?

10. What is the most important teens’ problem?

Задание 7. Переведите с английского предложения с сослагательным наклонением.

1.                 If it didn’t rain, we would go for a walk.

2.                 If we had a camera, we could take pictures of the beauti­ful scenery.

3.                 If there were any sugar left, we should not have to go to the shop.

4.                 If I knew him, I should ask his advice.

5.                 If you did not have a toothache, you would enjoy the party.

6.                 If you were not so absent-mind­ed, you would not make so many mistakes.

7.                 If you rang me up, I should know you were in trouble.

8.                 If you watched the cat, it wouldn’t eat the fish.

9.                 If it were not so late, we would go to see them.

10.            If I were you, I would read the book.

11.If it hadn’t rained yesterday, we would have gone for a walk.

12If we had had a camera during our trip to the USA, we could have taken pictures of the beauti­ful scenery.

13. If there had been any sugar left, we would not have gone to the shop late at night.

14. If you had not had a toothache last night, you would have enjoyed the party.

15.            If you had not been so absent-mind­ed at the last lesson, you would not have made so many mistakes in your test.

16.            If you had rung me up yesterday, I should have known you were in trouble.

17.            If you had watched the cat, it wouldn’t have eaten the fish.

18.            If it hadn’t been so late, we would have gone to see them.

19.            If I had had more time, I would have done the test better.

20.            If you hadn’t left the child alone, she would have broken the vase.

Задание 8. Раскройте скобки, используя сослагательное наклонение (третье условие)

1.                 If he hadn’t broken his bicycle, he would … (go) to the country.

2.                 If I hadn’t had a bad headache yesterday, I could (come) to see you.

3.                 If the ship … (not sail) near the coast, it would not have struck a rock.

4.                 If he … (be) in town, he would have been present at our meeting.

5.                 If the road … (not be) so slippery, I … (not fall) and hurt my leg.

6.                 If they .. (make) a fire, the wolves  … (run) away.

7.                 If I  … (expect) my friend to come, I … (not go) to the cinema.

8.                 If I … (have) a dictionary, I … (translate) the article yesterday.

9.                 If we … (get) a letter from him, we … (not worry).

10.            If she … (come) home late last night, her father … (be) angry.

Задание 9. Переведите следующие предложения в повелительном наклонении на английский язык.

1. Откройте ваши книги.
2. Подожди меня, пожалуйста.
3. Выключи музыку.
4. Скажи ему правду.
5. Давай закончим это.
6. Не плачь.
7. Обязательно посмотри этот фильм.

Критерии оценки:

5 «отлично» -  Правильное выполнение заданий. Студент отлично знает и применяет на практике грамматический материал по теме Сослагательное и повелительное наклонение в английском языке. Студент владеет необходимой  лексикой по теме Проблемы молодежи. Виды развлечений современной молодёжи, обладает навыками чтения и литературного перевода.

4 «хорошо» - Правильное выполнение 8 заданий из 9. Студент хорошо знает и применяет на практике грамматический материал по теме Сослагательное и повелительное наклонение в английском языке. Студент владеет необходимой  лексикой по теме Проблемы молодежи. Виды развлечений современной молодёжи, обладает навыками чтения и литературного перевода. Допускаются неточности при выполнении заданий.

3 «удовлетворительно» - Допускаются неточности и незначительные ошибки в выполнении  заданий. Студент не достаточно хорошо знает и применяет на практике грамматический материал по теме Сослагательное и повелительное наклонение в английском языке. Студент не в полной мере владеет необходимой  лексикой по теме Проблемы молодежи. Виды развлечений современной молодёжи, обладает навыками чтения и литературного перевода. Объем правильно выполненных заданий составляет 6 заданий.

2 «неудовлетворительно» - Отсутствует выполнение заданий. Студент не знает и не применяет на практике грамматический материал по теме Сослагательное и повелительное наклонение в английском языке. Студент не владеет необходимой  лексикой по теме Проблемы молодежи. Виды развлечений современной молодёжи, не обладает навыками чтения и литературного перевода. Объем правильно выполненных заданий составляет менее 4 заданий из 9.

Раздел 2. Тема 2.1Цифры, числа, математические действия.Условные предложения трёх типов.

Проверяемые результаты обучения: владение навыками чтения и перевода, знание особенностей условных предложений трех типов, умение распознавать и употреблять их в речи.

Задания (типовые) для оценки освоения раздела 2, темы 2.1Цифры, числа, математические действия.Условные предложения трёх типов.

Задание 1. Ознакомьтесь с количественными и порядковыми числительными, десятками и составными числительными.

 Числительные обозначают количество предметов или порядок предметов при счете.

Числительные делятся на количественные, отвечающие на вопрос сколько? и порядковые, отвечающие на вопрос который?

 Количественные числительные от 13 до 19 образуются прибавлением суффикса -teen к основе.

 Числительные, обозначающие десятки, имеют суффикс -ty. Порядковые числительные, кроме первых трех (first, second, third), образуются прибавлением суффикса -th или -eth к соответствующим количественным числительным.

Они всегда употребляются с определенным артиклем.

 Количественные числительные               Порядковые числительные

1 one — один                                            the first — первый

2 two — два                                               the second — второй

3 three — три                                             the third — третий

4 four                                                         the fourth

5 five                                                          the fifth

6 six                                                           the sixth

7 seven                                                       the seventh

8 eight                                                        the eighth

9 nine                                                         the ninth

10 ten                                                         the tenth

11 eleven                                                    the eleventh

12 twelve                                                   the twelfth

13 thirteen                                                  the thirteenth

14 fourteen                                                 the fourteenth

15 fifteen                                                    the fifteenth

16 sixteen                                                   the sixteenth

17 seventeen                                               the seventeenth

18 eighteen                                                 the eighteenth

19 nineteen                                                 the nineteenth

20 twenty                                                   the twentieth

Десятки:

Составные числительные:

20 twenty — the twentieth

twenty-two—the twenty-second

30 thirty — the thirtieth

thirty-three — the thirty-third

40 forty — the fortieth

forty-four— the forty-fourth

50 fifty — the fiftieth

fifty-five — the fifty-fifth

60 sixty — the sixtieth

sixty-six — the sixty-sixth

70 seventy — the seventieth

 

80 eighty — the eightieth

 

90 ninety — the ninetieth

 

Задание 2. Напишите цифрами следующие даты:

a) The first of March nineteen seventy-six.

b) The fifth of December two thousand.

c) The sixteenth of May nineteen five.

d) The third of July nineteen hundred,

in (the year) nineteen ninety-seven

in (the year) nineteen hundred eighty-one

in (the year) two thousand five.

Задание 3. Напишите по-английски:

1) 7 марта 1999 года;

2) 1 сентября 1974 года;

3) 22 апреля 1911 года;

4) 11 марта'1951 года;

5) 12 декабря 2024 года.

Как читаются дробные числительные?

Простые                       Десятичные

1/2 —a (one) half;          0.1 — О [ou] point one

1/4  -  a (one) quarter      2.4 5 — two Point four five

2/3 — two thirds            35. 25 — three five ( или : thirty-five) point two five

1.5  — one and a half

Задание 4. Напишите цифрами дробные числа:

Простые:

1) A (one) half; 2) two thirds; 3) a (one) quarter; 4) three fourths; 5) two and a (one) half; 6) five and one sixth; 7) a (one) fifth.

Десятичные:

1) Zero (nought/ou) point two; 2) two point four five; 3) four point five; 4) three four (thirty four) point one.

Задание 5. Прочтите по-английски:

1. Количественные числительные:

 3, 5,11, 12,13, 24, 69, 325, 1005, 530425, 1,745.033.

2. Порядковые числительные: 1, 2, 15, 23, 84, 149, 150, 208, 1000, 2.000.000.

Задание 6. Прочитайте диалог, переведите, обращая внимание на слова данные ниже.

Lines and Shapes

Teacher: Look at these simple figures. What’s this, Nick?

Nick: This is a point.

Teacher: That’s right. And what’s this, Jenny?

Jenny: This is a straight line.

Teacher: Correct. And what are these, Ann?

Ann: They are lines.

Teacher: What kind of lines?

Ann: I don’t know.

Teacher: Sue, what kind of lines?

Sue: They are parallel lines.

Teacher: Very good, Sue. Ann, please repeat, “These are parallel lines.”

Ann: These are parallel lines.

Teacher: Good. Are these two lines parallel, Tom?

Tom: No, they are not parallel. They are perpendicular.

Ann: What’s “perpendicular”?

Teacher: Perpendicular means “at right angle to another line or to a plane”. Now, what’s No 5? Who can answer my question?

Nick: I can. This is a curve.

Teacher: Quite correct. It’s a curve.

Now let’s look at these shapes or geometrical figures. What are they called?

Jenny: That’s simple. The first shape is a square, the second is a triangle, the third is a rectangle and the fourth is a circle.

Teacher: Right. So, what’s this, Ann?

Ann: It’s a triangle.

Teacher: Triangles have three sides. And what about squares? Ann?

Ann: They have four sides.

Teacher: Correct.

And their sides are equal. Is it also true about rectangles, Mary?

Mary: No, it’s not. Their opposite sides are equal and parallel.

Teacher: Very good, Mary.

Triangles, squares and rectangles have straight sides. They are rectilinear shapes.

And finally, look at these solid shapes. Shape No. 1 (number one) is a pyramid, No. 2 is a cylinder, No. 3 is a cube, and No. 4 is a cone.

A pyramid, a cylinder, a cube and a cone are solid figures.

That’s enough for today.

Your homework for tomorrow. Please read about angles in your textbook: obtuse angles, acute angles, and right angles.

John: What page, Miss?

Teacher: Page fifteen. The lesson is over. A break.

Vocabulary


figure  1. диаграмма, рисунок, чертеж; 2. цифра

point /point/ точка

straight line /.streit 'lain/ прямая линия

correct /ka'rekt/ правильный, верный

parallel lines /.paeralal 'lainz/ параллельные линии

perpendicular /,p3:pan'dikjola/ перпендикулярный

angle /'aeggl/ угол; at right angle to под прямым углом к

plane /plein/ плоскость

curve /кз.-v/ кривая

shape /feip/ фигура

square /skwea/ квадрат

triangletraiaeggl/ треугольник

side /said/ сторона

equal /'i:kwal/ равный

trueверный

rectangle /'rektaeggl/ прямоугольник

opposite /‘opazit/ противоположный rectilinear /.rekti'lina/ прямолинейный

finally /‘famali/ наконец

solid shapes /.solid 'jeips/ пространственные фигуры

number /'плтЬэ/ номер, число

pyramid /'piramid/ пирамида

cylinder /’sihnda/ цилиндр

cube /kju:b/ куб

cone /кэоп/ конус

enough /1'пд[/ достаточно

homework /'haumw3:k/ домашнее задание

textbook /'tekstbok/ учебник

obtuse /ab'tjuis/ angle тупой угол

acute /a'kju:t/ angle острый угол right /rait/ angle прямой угол

page /peidj/ страница

Miss /mis/ мисс, незамужняя жен. форма обращения учеников младших классов к учительнице в Англии



Задание 7. Переведите на русский язык следующие  условные предложения:

1. If I came later I would be late for the lesson. 2. If he had known the time-table he wouldn't have missed the train. 3. It would be better if you learned the oral topics. 3.1 wish I had known this before the examination. 4. I would have come to you if you had not lived so far away. 5. If I had seen you yesterday I would have given you my text-book. 6. If I were in your place I wouldn't buy the tickets beforehand. 7. If I had known that you needed help I would have helped you.

Задание 8. Раскройте скобки, употребляя глаголы в требующейся форме, определив тип условного предложения.

1. If he were not such an outstanding actor, he (not to have) so many admirers. 2. If you (to give) me your address, I shall write you a letter. 3. If she (not to be) so absent-minded, she would be a much better student. 4. If my sister does not go to the south, we (to spend) the summer in St. Petersburg together. 5. If they (not to go) to Moscow last year, they would not have heard that famous musician. 6. If you (not to get) tickets for the Philharmonic, we shall stay at home. 7. If you were not so careless about your health, you (to consult) the doctor. 8. I should be delighted if I (to have) such a beautiful fur coat. 9. If it (to rain), we shall have to stay at home. 10. If he (to work) hard, he would have achieved great progress. 11. If it is not too cold, I (not to put) on my coat. 12. I (to write) the composition long ago if you had not disturbed me. 13. If he (not to read) so much, he would not be so clever. 14. If my friend (to be) at home, he will tell us what to do.

Задание 9. Переведите следующие бессоюзные условные предложения.

1.                 Had I more time, I should get all that work finished long ago.

2.                 Had we waited to carry the canoe, time would have been lost.

3.                 Were the wire of smaller diameter, its resistance would be increased.

4.                 Should we want to accelerate the motion, we should have to apply some force.

Критерии оценки:

5 «отлично» -  Правильное выполнение заданий. Студент отлично знает и применяет на практике грамматический материал по теме Условные предложения трёх типов. Студент владеет необходимой  лексикой по теме Цифры, числа, математические действия, обладает навыками чтения и литературного перевода.

4 «хорошо» - Правильное выполнение 8 заданий из 9. Студент хорошо знает и применяет на практике грамматический материал по теме Условные предложения трёх типов. Студент владеет необходимой  лексикой по теме Цифры, числа, математические действия, обладает навыками чтения и литературного перевода. Допускаются неточности при выполнении заданий.

3 «удовлетворительно» - Допускаются неточности и незначительные ошибки в выполнении  заданий. Студент не достаточно хорошо знает и применяет на практике грамматический материал по теме Условные предложения трёх типов. Студент не в полной мере владеет необходимой  лексикой по теме Цифры, числа, математические действия, обладает навыками чтения и литературного перевода. Объем правильно выполненных заданий составляет 6 заданий.

2 «неудовлетворительно» - Отсутствует выполнение заданий. Студент не знает и не применяет на практике грамматический материал по теме Условные предложения трёх типов. Студент не владеет необходимой  лексикой по теме Цифры, числа, математические действия, не обладает навыками чтения и литературного перевода. Объем правильно выполненных заданий составляет менее 4 заданий из 9.

Раздел 2. Тема 2.2Основные геометрические понятия и физические явления. Инфинитив, его формы, инфинитивные обороты.

Проверяемые результаты обучения: владение навыками чтения и перевода, знание лексики по теме геометрические понятия и физические явления,форм инфинитива, особенностей инфинитивных оборотов Сложное дополнение и Сложное подлежащее, умение распознавать и употреблять их в речи.

Задания (типовые) для оценки освоения раздела 2, темы 2.2 Основные геометрические понятия и физические явления. Инфинитив, его формы, инфинитивные обороты.

Задание 1. Прочитайте текст, переведите.

Lines and Angles

A line in geometry is always a straight line. When two straight lines meet at a point, they form an angle. The lines are called sides or rays of the angle, and the point is called the vertex. The symbol for angle is .

Angles are usually measured in degrees. An angle of 30 degrees, written 30°, is an angle whose measurement is 30 degrees.

When two lines intersect, four angles are formed. The angles opposite each other are called vertical angles and are equal to each other.


a and c are vertical angles. a=c

b and d are vertical angles. b =d


A straight angle has its sides lying along a straight line. It is always equal to 180°.


ABC =B= 180°

В is a straight angle.


 

If the sum of the angles is a straight angle (180°), the two angles are supplementary and each angle is the supplement of the other.

 


G is a straight angle = 180°.

a + b= 180°

a and b are supplementary angles.


If two supplementary angles are equal, they are both right angles. A right angle is a one-half of a straight angle. Its measureis 90°.


 

 


Задание 2. Ознакомьтесь с содержанием текста, выпишите определения с переводом.

GEOMETRICPROGRESSION.POWERS AND ROOTS

An Indian legend says: prince Sirahm was so pleased with the invention of chess, that he offered the inventor of chess any reward he might wish. The inventor asked 1 wheat seed for the first square of the chess-board, 2 seeds for the second square, 4 for the third and so on, doubling the number of seeds for every subsequent square. The prince agreed. But the quantity of wheat gathered from all the country was far from the required number. The prince got angry, he never thought his land to be so poor not to be able to satisfy such a modest wish. Mathematicians were called to help.

The quantity of wheat for every square on the board was calculated and it was discovered that the inventor couldn’t receive his reward because of the lack of wheat.

Now let us count with them how much wheat had to be given to the inventor. There are 64 squares on the chess-board.

The number of seeds for the 64 squares is equal to the sum S of the following series of numbers:

S = 1 + 2 + 22 + 23 + 24 + ... + 262 + 263.

After the transformations of the equality we receive the formula S=264—1 to solve which it is necessary to find the power of 264.

It may be done by multiplication by 2 according to the formula:

264 = 2 • 2 • 2 • 2 (64 factors), or 264 = [(216)2]2 = (65,5362)2

and the final number of seeds according to the formula: S = 264 — 1 will be equal 18,446,744,073,709,551,615. This enormous number of seeds spread over the surface of the earth would form a layer of wheat of 9 mm thick.

Such a series of numbers each term of which, beginning with the second, is equal to the product of the preceding one and some number constant for this series is called a geometric progression.

When a number is used a certain number of times as a factor, the product is called a power of the number.

When 2 is used twice as a factor the product is the second power of 2 or the square of 2, if used three times then the product is the third power, or the cube, four times-the fourth power of 2, n times or any number of times the nth power of 2.

x3 (x cube) means x • x • x. The number 3 is called an index or an exponent of the power, the letter x is the base of the power. 33=3 • 3 • 3=27. This equality expresses an operation known in mathematics as involution.

The operation by means of which the base may be found according to a given power and the exponent of that power is called evolution and is expressed as follows: 49. i.e. the square root of 49. The square root of 49 is 7, since 72 = 49 an (- 7)2 = 49.

The root of the third degree (or cube root) of 64 is 4, since 43 =4 • 4 • 4=64. The number n, indicating the degree of a root is called the index of the root. The symbol is called the radical sign or the sign of the root.

The operation by means of which the exponent of a power may be found according to a given power and a given base is called finding the logarithm of the given number.

Задание 3.  Вставьте частицу to перед инфинитивом, где это необходимо.

1.     Parents should let the children … have private life.

2.     I waited for my friend … get off the bus.

3.     They made me … do it.

4.     We can’t … let him stay outdoors.

5.     He ordered the car … come at 5 p.m.

6.     At last he was made … write a letter to his parents.

7.     Children were allowed … go to the cinema alone.

8.     Let us … be friends.

9.     What makes you … think so?

10. Don’t let him … drive so fast.

Задание 4. Напишите все возможные формы инфинитива глаголов: to do, to read, to ask, to talk, to sing, to learn.

ПРИМЕР to write

Active Infinitive: to be writing, to have written, to be writing, to have been writing

Passive Infinitive: to be written, to have been written

Задание 5. Раскройте скобки, употребляя требующуюся фор­му инфинитива.

1. Не seems (to read) a lot. 2. Не seems (to read) now. 3. He seems (to read) since morning. 4. He seems (to read) all the books in the library. 5. I want (to take) you to the concert. 6. I want (to take) to the concert by my father. 7. She was glad (to help) her friends. 8. She hoped (to help) by her friends. 9. I hope (to see) you soon. 10. We expect (to be) back in two days. 11. He expected (to help) by the teacher. 12. The children seem (to play) since morning. 13. I am glad (to do) all the homework yesterday.14. She seems (to work) at this problem ever since she came here. 15. I am sorry (to break) your cup. 16. His English seems (to get) better. 17. He is glad (to help) with his health problems.

Задание 6. Переведите на английский язык, употребляя требующуюся форму инфинитива.

1. Я рад, что рассказал вам эту историю. 2. Я рад, что мне рассказали эту историю. 3. Я хочу по­знакомить вас с этой артисткой. 4. Я хочу, чтобы меня познакомили с этой артисткой. 5. Я рад, что встретил ее на станции. 6. Я рад, что меня встретили на станции. 7. Мы очень счастливы, что пригласили его на вечер. 8. Мы очень счастливы, что нас пригласили на вечер. 9. Он будет счаст­лив посетить эту знаменитую картинную галерею. 10. Он был счастлив, что посетил эту знаменитую картинную галерею. 11. Дети любят, когда им рассказывают сказки. 12. Мне очень жаль, что я пропу­стил эту лекцию. 13. Она счастлива, что была на этом концерте. 14. Она рада, что присутствовала на лекции. 15. Он очень доволен, что выиграл кубок. 16. Он был счастлив, что снова дома. 17. Он был счастлив, что вернулся домой. 18. Я сожалею, что прервал вас.

Задание 7. Переведите на английский язык, употребляя инфинитивный оборот сложное дополнение.

1. Он услышал, что кто-то зовет его по имени. 2. Мама хотела, чтобы я полила цветы. 3. Я видел, как она вышла из трамвая и перешла улицу. 4. Яне ожидал, что моя сестра получит плохую оценку. 5. Я знаю, что ваш друг — футболист-профессионал. 6. Я хочу, чтобы это правило выучили. 7. Я слышал, как кто-то постучал в дверь. 8. Когда ты починишь свои часы? 9. Я знаю, что ваш брат болен. 10. Мама заставила меня поехать за город (на дачу). 11. Его родители не рассчитывали, что после школы он пойдет в университет. 12. Я заставлю его принести книгу завтра. 13. Я рассчитываю, что он мне позвонит. 14. Мы хотели, чтобы они достигли успеха. 15. Мы бы не хотели, чтобы учитель подумал, что мы нарочно опоздали. 16. Он терпеть не мог, когда люди громко смеялись. 17. Я видела, как эти дети кормят уток в нашем парке. 18. Они не видели, как он приехал.

Задание 8. Перефразируйте следующие предложения, употребляя инфинитивный оборот сложное дополнение вместо придаточных дополнительных предложений.

1. I felt that somebody touched me lightly on the shoulder. 2. He heard that someone called his name. 3. They heard how the woman uttered a little exclamation. 4. I should like to see how he would say it to my face. 5. I expect that you will join our excursion. 6. We had not expected that she would reply, but she did. 7. We knew that he was a clever man. 8.1 don’t like it that you repeat this nonsense. 9.1 hate it when people speak so cynically. 10. We expect that everybody will be ready by seven. 11. They showed themselves even more narrow-minded than we had expected they would be. 12. We did not expect that he would return so soon. 13. Hehateditwhenpeoplearguedabouttrifles.


Задание 9. Перефразируйте следующие предложения, употребляя инфинитивный оборот сложное подлежащее.

• E.g. We heard that a car stopped outside the door. A car was heard to stop outside the door.

It is believed that the poem was written by Byron.

The poem is believed to have been written by Byron.

1. People consider the climate there to be very healthy. 2. It was announced that the Chinese dancers were arriving next week. 3. It is expected that the performance will be a success, 4. It is said that the book is popular with both old and young. 5. It is believed that the poem was written by an unknown soldier. 6. It is supposed that the well-known playwright is working on a new play. 7. It is reported that the flood has caused severe damage to the crops. 8. It was supposed that the crops would be rich that year. 9. It has been found that this mineral water is very good for the liver. 10. Scientistsconsiderthatelectricityexiststhroughout

 

Критерии оценки:

5 «отлично» -  Правильное выполнение заданий. Студент отлично знает и применяет на практике грамматический материал по теме Инфинитив, его формы, инфинитивные обороты.  Студент владеет необходимой  лексикой по теме Основные геометрические понятия и физические явления, обладает навыками чтения и литературного перевода.

4 «хорошо» - Правильное выполнение 8 заданий из 9. Студент хорошо знает и применяет на практике грамматический материал по теме Инфинитив, его формы, инфинитивные обороты. Студент владеет необходимой  лексикой по теме Основные геометрические понятия и физические явления, обладает навыками чтения и литературного перевода. Допускаются неточности при выполнении заданий.

3 «удовлетворительно» - Допускаются неточности и незначительные ошибки в выполнении  заданий. Студент не достаточно хорошо знает и применяет на практике грамматический материал по теме Инфинитив, его формы, инфинитивные обороты. Студент не в полной мере владеет необходимой  лексикой по теме Основные геометрические понятия и физические явления, обладает навыками чтения и литературного перевода. Объем правильно выполненных заданий составляет 6 заданий.

2 «неудовлетворительно» - Отсутствует выполнение заданий. Студент не знает и не применяет на практике грамматический материал по теме Инфинитив, его формы, инфинитивные обороты. Студент не владеет необходимой  лексикой по теме Основные геометрические понятия и физические явления, не обладает навыками чтения и литературного перевода. Объем правильно выполненных заданий составляет менее 4 заданий из 9.

            Раздел 2. Тема 2.3 Промышленность, транспорт; детали, механизмы. Герундий и герундиальные обороты.

Проверяемые результаты обучения: владение навыками чтения и перевода, знание форм герундия, специфики герундиальных оборотов, умение распознавать и употреблять их в речи.

Задания (типовые) для оценки освоения раздела 2, темы 2.3 Промышленность, транспорт; детали, механизмы. Герундий и герундиальные обороты.

Задание 1. Ознакомьтесь с содержанием текста, охарактеризуйте кратко машиностроение, химическую промышленность и легкую промышленность России.

Machine-building Industry

Russia’s machine-building industry provides1 most of the federation’s requirements2 for steam boilers and turbines, electric generators, grain combines, automobiles, and electric locomotives and fills much of its demand for machine tools, instruments, and automation components. Important automobile factories are located3 in Moscow, Nizhny Novgorod, Miass, Yaroslavl, Ulyanovsk (formerly Simbirsk), and Izhevsk; the largest plants include the Zhiguli works at Tolyatti (near Samara) and the heavy truck factory at Naberezhnye Chelny (in Tatarstan).

Chemical Industry

Because of the complex history of the development of the chemical industries and the great variety of raw materials4 involved5, chemical manufacture6 is widely dispersed7. The industry initially utilized mineral salts8, coke-oven9 and smelter10 gases, timber11 and foodstuffs12 (mainly potatoes) as their raw materials. On this basis, synthetic rubber15 factories were built in the Central Black Earth and Central regions, areas of large-scale potato production; sulfuric acid14 plants15 were developed in the Urals and North Caucasus, where there was nonferrous metallurgy16, and potassium and phosphatic fertilizer17 plants were constructed at sites in several regions, near deposits of potassium salts and phosphorites.

Since the end of the 1950s the massive increase in oil and gas output18 has provided new raw chemical materials and lessened the dependence19 on traditional resources. New chemical plants have been built both in the oil- and gas-producing areas of the Volga-Ural and North Caucasus zones and in other regions at points served by pipelines20. Chemical industries requiring large quantities of electric power, such as those based on cellulose, are particularly important in Siberia, where both timber and electricity are plentiful21.

Light Industry

Russia’s textile industries are heavily concentrated in the European sector, especially in the Central region, which produces a large share of the federation’s clothing and footwear22.

The dominant branch23 is cotton textiles24, with the raw cotton25 coming mainly from the Central Asian states. In the zone between the Volga and Oka rivers, east of Moscow, there are numerous cotton textile towns, the largest of which are Ivanovo, Kostroma, and Yaroslavl. Durable consumer goods26 – refrigerators27, washing machines28, radios and television sets, and the like29 - are produced primarily30 in areas with a tradition of skilled industry31, notably in and around Moscow and St. Petersburg.


1. обеспечивать

2.потребности

   3. расположены

4. сырьё

5. требующееся

6. химическое производство

7. разбрасывать, рассредоточивать

8. минеральные соли

9. коксовая печь

10. плавильный завод

11. строевой лес

12. пищевые продукты

13. синтетический каучук

14. серная кислота

15. заводы

16. цветная металлургия

17. калийные и фосфатные

18. удобрения

19. добыча

20. ослабило зависимость

21. трубопровод

22. имеются в изобилии

23. одежда и обувь

24. преобладающая отрасль

              25.хлопчатобумажное производство

26. хлопок-сырец

27. товары длительного пользования

28. холодильники

29.стиральные машины

 

30. преимущественно

31. отрасли промышленности, требующие квалифицированной рабочей силы


Задание 2. Прочитайте текст, предварительно ознакомившись со словами, составьте план и краткое изложение содержания текста.

Transport for Tomorrow

One thing is certain about the public transport of the future: it must be more efficient than it is today. The time is coming when it will be quicker to fly across the Atlantic to New York than to travel from home to office. The two main problems are: what vehicle shall we use and how can we plan our use of it?

There are already some modern vehicles which are not yet in common use, but which may become a usual means of transport in the future. One of these is the small electric car: we go out into the street, find an empty car, get into it, drive to our destination, get out and leave the car for the next person who comes along. In fact, there may be no need to drive these cars. With an automatic guidance system for cars being developed, it will be possible for us to select our destination just as today we select a telephone number, and our car will move automatically to the address we want.

For long journeys in private cars one can also use an automatic guidance system. Arriving at the motorway, a driver will select the lane1 he wishes to use, switch over to automatic driving, and then relax — dream, read the newspaper, have a meal, flirt with his passenger — while the car does the work for him. Unbelievable? It is already possible. Just as in many ships and aircraft today we are piloted automatically for the greater part of the journey, so in the future we can also have this luxury in our own cars.

A decade ago, the only thing electronic on most automobiles was the radio. But at present sophisticated electronics is playing a big part in current automotive research. For example, in every gasoline-powered2 car that General Motors Corporation makes there is a small computer continuously monitoring the exhaust. The device, about the size of a pack of cigarettes, adjusts the vehicle carburetor fuel intake3 to get the best fuel economy. Ford cars are equipped with an electronic instrument panel that, among other things4, will calculate how far one can drive on the fuel left in the tank. It will also estimate the time of arrival at destination and tell the driver what speed he has averaged5 since turning on the ignition.

According to specialists these features made possible by micro-electronics are only the beginning. Radar may control the brakes to avoid collisions, and a display screen may show the car’s position on the road. Recently a radar to be mounted on lorries and cars has been designed in the USA. The radar aerial looks like a third headlight placed directly above the bumper. Having summed up the in-formation about the speed and distance of various objects ahead, the computer detects all possible dangers and their nature. A third component in the system is a monitor on the instrument panel. The radar only observes objects ahead of the vehicle. It is automatically turned on when the speed exceeds ten miles an hour. The green light on the panel indicates that the system is on. The yellow light warns of stationary objects ahead, or something moving slower than the car. The red light and buzzer warn that the speed should go down. Another red light and sound signal make the driver apply the brakes.

A Japanese company is designing a car of a new generation. When completed, the new model will have a lot of unusual characteristics. The car’s four-wheel control system will ensure movement diagonally and even sideways, like a crab, at right angles to the longitudinal axis. This is especially important when leaving the car in parking places. To help the driver get information while concentrating on the road, the most important data will be projected on the wind screen. A tourist travelling in such a car will not lose his way even in Sahara with its impassable roads: a navigation Earth satellite will indicate the route.

A new ceramic engine has been developed in Japan. Many important parts as pistons, pressure rings6, valves and some othershave been made of various ceramic materials, piston rings7 made of silicon materials being in many respects better than those of steel. They withstand temperatures up to 1,000 °C. Therefore, the engine does not need a cooling system.

adjust  — регулировать

angle  — угол

apply  — применять

avoid  — избегать

axis  — ось

current — современный, текущий

destination  — пункт назначения

detect  — обнаруживать

directly  — прямо, непосредственно

engine  — двигатель

ensure  — обеспечивать, гарантировать

equip  — оборудовать

exceed  — превышать

exhaust  — выхлоп

guidance  — управление, наведение

ignition  — зажигание

indicate  — указывать, показывать

make  — делать, заставлять

mount  — монтировать, устанавливать

only  — единственный

place  — помещать

select  — выбирать

size n — размер

sophisticated  — сложный

valve  —клапан

warn  —предупреждать

withstand  — выдерживать

lane — ряд

gasoline - powered — с бензиновым двигателем

fuel intake — впрыск топлива

among other things — кроме всего прочего

what speed he has averaged — какова была его средняя скорость

pressure ring — уплотнительное кольцо

piston ring — поршневое кольцо

Задание 3. Просмотрите текст и ответьте на вопросы.

1. What is the text about? 2. What kind of a car may be in common use in the near future? 3. How will a public electric car operate? 4. How will it operate on a motorway? 5. What electronic devices are there in a modern car? 6. What electronic devices does General Motors Corporation offer for a car? 7. What electronic devices are Ford cars equipped with? 8. Can a radar be used in a car? What will its functions be? 9. What functions will a Japanese car of a new generation have? 10. What materials do the Japanese offer to use for car motors?

Задание 4. Укажите, какие из приведенных утверждений соответствуют содержанию текста.

1. An automatic guidance system was developed for the electric car. 2. Small electric cars are in common use. 3. Many ships and aircrafts are piloted automatically for the greater part of the journey. 4. Usually having arrived at a motorway, a driver switches over to automatic control and relaxes. 5. A decade ago there were many electronic things in the cars. 6. There is no future for microelectronics in automobiles. 7. Recently a radar to be mounted on lorries and cars has been designed in the USA. 8. A new ceramic engine has been developed in France.

Задание 5. Переведите на русский язык, обращая внимание на способы перевода герундия.

1.          He always suggested staying here.

2.          The job involves travelling to Germany once a month.

3.          I proposed having party at the beach.

4.          I promised to care for the cat but I’m not much good at babysitting.

5.          He is capable of standing on his head and playing the saxophone.

6.          You’d better start digging the garden.

7.          Writing letters is more boring than phoning.

8.          It is not worth helping him do this job.

9.          My wife apologized for being late.

10.     I’m very excited about attending tomorrow’s game.

Задание 6. Используйте в предложениях герундий.

1.          There is no sense in … (earn) more money than you can spend.

2.          Do you mind … (work) overtime?

3.          Normally I enjoy … (go) out but today I’d prefer … (stay) indoors.

4.          The film was really worth … (see).

5.          Brent is looking forward to … (take) a short break next month.

6.          She is fond of … (have) picnics.

1.          I can’t remember … (see) him before.

2.          Everybody enjoys … (work) with him.

3.          The boy hates … (scold).

4.          I am sorry for … (disturb) you.

5.          The windows need … (clean).

6.          It is no good … (force) him to go with us.

7.          I hate … (ask) stupid questions.

8.          I don’t like … (cheat).

Задание 7. Переведите предложения с русского на английский, используя герундий:

1.          Мой дядя бросил курить и сейчас предпочитает есть.

2.          Пожалуйста, прекратите шептаться.

3.          Мне нравится быть одному. Я никогда не чувствую себя одиноко.

4.          Я перешел дорогу, не посмотрев.

5.          Подумай хорошо (carefully), прежде чем принять решение.

6.          Попробуй нажать на кнопку!

7.          Она закончила красить свою квартиру.

8.          Ты можешь представить свою жизнь без ТВ?

9.          Я сожалею, что рассказал Джулии свой секрет; она рассказала всем.

10.     Он всех поблагодарил за то, что пришли.

Задание 8. Переведите предложения, определяя зависимыe и независимыe герундиальныe обороты. Обращайте внимание на предлоги, вводящие герундиаль­ные обороты.

1. John's coming so late surprised everybody2. Do you mind my joining the discussion? 3. The teacher insisted on our taking part in the conference. 4. Can you rely on your assistant doing it? 5. His coming back so soon was so unexpected. 6. She left her place at the sound of the key being turned in the lock7. They succeeded in getting reliable informa­tion on dealing with this type of error. 8. In spite of having met with failure they continued experimenting. 9. Metals cannot be dissolved without being changed into new substances. 10. He has the reputation of having been a man of piety. 11. They insisted on the sample being tested repeatedly. 12. They objected to using greater voltage in this case. 13. All aspects of life depend on our understanding the properties of matter. 14. We know of man's having learned to use wood, stone and metal in prehistoric times. 15. A comparatively simple act of driving a car requires a vast number of scientific principles. 

Задание 9. Ответьте на следующие вопросы.

Какие формы времени и залога имеет герундий?

В функции каких членов предложения употребляется герундий?

Чем определяется выбор способа перевода герундия?

Какие способы перевода герундия существуют?

Какие герундиальные обороты существуют?

Какие существуют способы перевода зависимого герундиального оборота?

Какие существуют способы перевода независимого герундиального оборота?

Критерии оценки:

5 «отлично» -  Правильное выполнение заданий. Студент отлично знает и применяет на практике грамматический материал по теме Герундий и герундиальные обороты.Студент владеет необходимой  лексикой по темеПромышленность, транспорт; детали, механизмы,обладает навыками чтения и литературного перевода.

4 «хорошо» - Правильное выполнение 8 заданий из 9. Студент хорошо знает и применяет на практике грамматический материал по теме Герундий и герундиальные обороты. Студент владеет необходимой  лексикой по теме Промышленность, транспорт; детали, механизмы, обладает навыками чтения и литературного перевода. Допускаются неточности при выполнении заданий.

3 «удовлетворительно» - Допускаются неточности и незначительные ошибки в выполнении  заданий. Студент не достаточно хорошо знает и применяет на практике грамматический материал по теме Герундий и герундиальные обороты. Студент не в полной мере владеет необходимой  лексикой по теме Промышленность, транспорт; детали, механизмы, обладает навыками чтения и литературного перевода. Объем правильно выполненных заданий составляет 6 заданий.

2 «неудовлетворительно» - Отсутствует выполнение заданий. Студент не знает и не применяет на практике грамматический материал по теме Герундий и герундиальные обороты. Студент не владеет необходимой  лексикой по теме Промышленность, транспорт; детали, механизмы, не обладает навыками чтения и литературного перевода. Объем правильно выполненных заданий составляет менее 4 заданий из 9.

 

Раздел 2. Тема 2.4 Оборудование, работа. Причастие, его формы, отличие причастия от герундия.

Проверяемые результаты обучения: владение навыками чтения и перевода, знание форм причастия, особенностей причастных оборотов, отличий причастия от герундия.

Задания (типовые) для оценки освоения раздела 2, темы 2.4 Оборудование, работа. Причастие, его формы, отличие причастия от герундия.

Задание 1. Прочитайте текст Основные виды оборудования и особенности его ремонта, найдите в тексте английские эквиваленты предложений, данных ниже, запишите.

Main types of equipment and features of its repair.

Equipment for technological purposes is divided into a number of types: metal-cutting machines, forging and pressing equipment, woodworking, foundry, lifting and transport equipment, etc.

Each of the types of equipment on the same basis is divided into groups, for example: metal-cutting machines-turning, drilling, milling, etc.; forging and pressing equipment - hammers, presses, scissors, etc.

Groups in accordance with the design are divided into types and sizes, for example: lathes are divided into screw-cutting lathes, turning-revolver lathes, turning-carousel lathes, etc.

The weight of the equipment is one of the essential parameters that affect the complexity of repairs. When the weight of the equipment changes, the ratio of labor costs and materials for repairs significantly changes. So depending on the mass of the equipment is divided into categories: easy — t, medium — 10, large — up to 30, hard — 60, very hard — 100, unique — over 100 tons In addition, equipment weighing up to a ton is considered transportable, and over a ton is considered non-transportable.

It is advisable to deliver transportable equipment for major repairs to specialized repair plants (SRZ) or to specialized mechanical repair shops (SRMC) of production associations. Specialized repair of non-transportable equipment is economically feasible to produce field repair teams.

According to the degree of automation, all types of equipment are divided into varieties:

- an assembly (machine) with manual control — this is the name of equipment that needs at least a worker's command to perform each individual (working or idle) movement, which makes up the part processing cycle;

- a semi-automatic machine is an assembly that performs the entire processing cycle of a part automatically and requires the intervention of a worker only for initial adjustment, installation and fixing of work pieces, removal of spent parts and commands to repeat the processing cycle;

- an automatic machine is an assembly that performs all the working and idle movements of the processing cycle of a batch of parts (including the removal of processed parts, installation and fixing of new work pieces) and needs only adjustment;

- the unit (machine, machine) with program control is a semi-automatic or automatic machine, which is controlled according to a pre-compiled and easily replaceable program.

Automatic machines can be connected in automatic lines.

An automatic line is a complex of units connected by a transport system and connected by electric, hydro and pneumatic automation systems.

A section of an automatic line is one or more units that are part of an automatic line, connected to other sections by means of storage devices.

The higher the degree of automation of equipment, the higher the requirements for compliance with the rules of technical operation, timeliness and quality of maintenance and repair.

Therefore, increasing the degree of automation leads to an increase in labor costs for maintenance and repair, as well as to an increase in the qualification complexity of repair and maintenance of equipment.

Depending on the accuracy characteristics, the equipment is divided into five accuracy classes: normal — H, increased-P, high-B, especially high-A, special-C.

High-precision machines P provide processing accuracy on average within 0.6 of the permissible deviations obtained on machines of normal accuracy.

High-precision machines B provide processing accuracy within 0.4, and especially high-precision A-within 0.25 of the permissible deviations obtained on machines of normal accuracy.

Especially precise machines C are designed to achieve the highest accuracy and are used for the final processing of parts such as dividing wheels and discs, reference wheels, measuring screws, etc. The permissible deviations on these machines do not exceed 0.16 of the normalized accuracy for machines.

Classification by accuracy, on the one hand, is necessary to establish increased requirements for the accuracy of manufacturing replacement parts and assembly when repairing machines of classes B, A and C. On the other hand, it is necessary to assess the complexity of their repair.

Find English equivalents to these sentences.

1) Оборудование по технологическому назначению разделяют на ряд видов: металлорежущие станки, кузнечно-прессовое оборудование, деревообрабатывающее, литейное, подъемно-транспортное оборудование и т. д.

2) Масса оборудования — один из существенных параметров, влияющих на трудоемкость ремонта.

3)Транспортабельное оборудование целесообразно доставлять для капитального ремонта на специализированные ремонтные заводы (СРЗ) или в специализированные ремонтно-механические цехи (СРМЦ) производственных объединений.

4) По степени автоматизации все виды оборудования делят на разновидности:
— агрегат (станок, машина) с ручным управлением — так называют оборудование, которое для выполнения каждого отдельного (рабочего или холостого) движения, из которых слагается цикл обработки детали, нуждается хотя бы в команде рабочего;
— полуавтомат — это агрегат, выполняющий весь цикл обработки детали автоматически и требующий вмешательства рабочего лишь для первоначальной наладки, установки и закрепления заготовок, снятия отработанных деталей и команд на повторение цикла обработки;
— автомат — это агрегат, выполняющий все рабочие и холостые движения цикла обработки партии деталей (включая снятие обработанных деталей, установку и закрепление новых заготовок) и нуждающийся лишь в наладке;
         5) Чем выше степень автоматизации оборудования, тем более высокие требования предъявляются к выполнению правил технической эксплуатации, своевременности и качеству выполнения технического обслуживания и ремонта.

6) Поэтому повышение степени автоматизации приводит к увеличению затрат труда на техническое обслуживание и ремонт, а также к увеличению квалификационной сложности ремонта и технического обслуживания оборудования.

Задание 2. Прочитайте текст Типовой комплект оборудования для лаборатории «Экологический практикум», найдите в тексте английские эквиваленты для предложений, данных ниже, переведите оставшиеся предложения.

Equipment for the laboratory "Ecological Workshop"

The standard set of equipment for the laboratory "Ecological Workshop" is intended for conducting educational and research works and workshops of environmental orientation on the topic of environmental assessment of the state of the environment in general secondary education (chemistry, ecology, biology, electives, specialized courses), in professional and additional education.

Creates maximum opportunities for involving students in a variety of activities, implementing training methods on relevant environmental topics.

Object of research water, air, hydrobionts, health indicators, soil, food, radiation, salts and bulk media

Allows to carry out demonstration experiments and laboratory front (student experiments) on the topics "Water", "Air", "Soil", "environment and health", to provide work in terms of audience, educational laboratories, in the field, to conduct basic and advanced levels within the classroom, laboratories and in the field; to implement creative and design work on the assessment of the environmental, health, and environmental security.

Allows you to evaluate environmental indicators:

- water-organoleptic parameters, turbidity, transparency, pH, total iron, mineral composition (sulfates, carbonates, bicarbonates, chlorides), nitrates, total hardness, dissolved oxygen, chromates, potassium and sodium (sum), active chlorine, sulfides, color, alkalinity;

- air environment-carbon dioxide, ammonia, dust;

- soil-acidity, salinity (content of soluble salts), mechanical composition,

- food products (vegetables, fruits, juices) - nitrates, acidity;

- health indicators - by the activity of saliva enzymes (the effect of smoking, antibiotics, acidity).

Find English equivalents to these sentences.

1) Типовой комплект оборудования для лаборатории «Экологический практикум» предназначен для проведения учебно-исследовательских работ и практикумов экологической направленности по тематике экологической оценки состояния окружающей среды в общем среднем образовании (химия, экология, биология, факультативы, профильные курсы), в профессиональном и дополнительном образовании.

2) Объект исследований вода, воздух, гидробионты, показатели здоровья, почва, продукты питания, радиация, соли и сыпучие среды
3) Позволяет оценивать показатели окружающей среды: воды – органолептические показатели, мутность, прозрачность, рН, железо общее, минеральный состав (сульфаты, карбонаты, гидрокарбонаты, хлориды), нитраты, общая жесткость, растворенный кислород, хроматы, калий и натрий (сумма), активный хлор, сульфиды, цветность, щелочность;

Задание 3. Переведите предложения с причастиями на русский язык.

1. A letter sent from St. Petersburg today will be in Moscow tomorrow.

2. Some of questions put to the lecturer yesterday were very important.

3. The girl putting the book on the shelf is the new librarian.

4. While putting the eggs into the basket, she broke one of them.

5. A fish taken out of the water cannot live.

6. A person taking a sunbath must be very careful.

7. Having taken a dictionary, he began to translate the text.

8. A line seen though this crystal looks double.

9. A teacher seeing a mistake in a student’s dictation always corrects it.

Задание 4. Переведите на английский язык.

1. приносящий; принесенный; принося; принеся; приносимый; будучи принесенным;

2. переводящий; переведенный; переводя; переведя; переводимый; будучи переведенным;

3. дающий; данный; давая; дав; даваемый; будучи данным;

4. делающий; сделанный; делая; сделав; делаемый; будучи сделанными;

5. говорящий; сказанный; говоря; сказав; будучи сказанным; произносимое;

6. играющий; сыгранный; играя; сыграв; играющийся; будучи сыгранным;

Задание 5. Выберите нужную форму причастия.

1.

a) We listened to the girls (singing, sung) Russian folk songs.

b) We listened to the Russian folk songs (singing, sung) by the girls.

2.

a) The girl (washing, washed) the floor is my sister.

b) The floor (washing washed) by Helen looked very clean.

3.

a) Who is that boy (doing, done) his homework?

b) The exercise (doing, done) by the pupils were easy.

4.

a) The house (surrounding, surrounded) by tall trees is very beautiful.

b) The wall (surrounding, surrounded) the house was very high.

5.

a) The girl (writing, written) is our best pupil.

b) Everything (writing, written) here is quite right.

6.

a) Read the (translating, translated) sentences one more.

b) The pupils (translating, translated) a very difficult text are tired.

Задание 6. Заполните пропуски в предложениях, причастиями, требуемыми по смыслу.

1. Девочка, зовущая меня, моя сестра.

The girl … me is my sister.

2. Он смотрел на женщину, сидящую за столом.

He looked at the woman … at the table.

3. Текст, переведенныйучеником, - легкий.

The text … by the pupil is easy.

4. Идя в школу, я встретил дядю.

… to school, I met my uncle.

5. Они увидели бегущих спортсменов.

They saw … sportsmen.

6. Написав письмо, он пошел на работу.

… the letter, he went to work.

7. Он стоял, смотря игру.

He stood … the game.

8. Закончив работу, он пошел домой.

… the work, he went home.

9. Уходя на работу, он забыл сумку.

… for work, he forgot to take his bag.

10. Это статья, описывающая политические события.

This is an article … the political events.

Задание 7. Распределите в 2 колонки: a) предложения, в которых ing-форма является причастием; b) предложения, в которых ing-форма является герундием, письменно переведите.

1. Не was looking at the plane flying over head. 2. Wishing to learn to skate, she bought herself a pair of skates. 3. Justimagine his coming first in the race! 4. The children were tired of running. 5. Being frightened by the dog, the cat climbed a high fence. 6. It is no use going there now. 7. Coming out of the wood, the travelers saw a ruined castle in the distance. 8. My greatest pleasure is travelling. 9. Growing tomatoes need a lot of sunshine. 10. Growing corn on his desert island, Robinson Crusoe hoped to eat bread one day. 11. Growing roses takes a lot of care and attention. 12. Having prepared all the necessary equipment, they began the experiment. 13. Mary will stop for a few days at the seaside before going back home. 14. While translating the text, I looked up many words in the dictionary. 15. I usually help mother by washing the dishes and doing the rooms. 16. Entering the room, I saw my friends smiling at me. 17. Instead of phoning his friend, he went to see him. 18. The boys continued playing football. 19. Watching the playing kittens was great fun for the children.

Задание 8. Выпишите предложения,в которых ing-форма является причастием.

1. The driving wheel of the machine is broken. 2. Driving in a motor car, we passed many villages. 3. We have every chance of passing our examinations well. 4. Having been knocked down by a passing car, the poor man was at once taken to hospital. 5. You don't know what you miss, not having the desire to listen to good music. 6. These happenings are remarkable. 7. Travelling is a pleasant way of improving one's education. 8. Every trust arranges for the marketing of its products. 9. I was told of a great friendship existing between the two captains. 10. It is no use crying over spilt milk. 11. Asking him for help is useless. 12. Happily we escaped being delayed on our way. 13. I stopped knocking at the door and, sitting down at the top of the stairs, began waiting for my father to come.14. Having stopped crying, the child quieted down to hard thinking. 15. Tom lived there like a paying guest, attracting very little attention of the others.

Задание 9. Соотнесите английские предложения с зависимыми причастными оборотами с их русскими эквивалентами.

1.                 Theideacanbepronounced true if tested by experience.

2.                 Mercury is used in barometers, having a high specific gravity.

3.                 Deeply shocked, I decided never to speak to her again.

4.                 If desired the instrument may be used repeatedly.

5.                 Not knowing what to do, I telephoned the police.

a)                 Так как я был глубоко возмущён, я решил больше никогда не разговаривать с ней.

b)                Эта идея может быть объявлена правильной, если она проверена на опыте.

c)                 Так как я не знал, что делать, я позвонил в полицию.

d)                Ртуть используется в барометрах, так как имеет большой удельный вес.

e)                 Если нужно, этот прибор может быть использован многократно.

Соотнесите английские предложения с абсолютными независимыми причастными оборотами с их русскими эквивалентами.

1.                 Nobody having any more to say, the meeting was closed.

2.                 All the money having been spent, we started looking for work.

3.                 One link broken, the whole chain is broken.

4.                 With this value constant, we could calculate the result.

5.                 Performance observations were recorded, with particular attention on the variables.

a)                 Наблюдения за работой (механизма) регистрировались, причём особое внимание уделялось переменным величинам.

b)                Так как никому больше нечего было сказать, собрание было закрыто.

c)                 Если разрушено одно звено в цепи, то разрушается вся цепь.

d)                После того, как все деньги были истрачены, мы начали искать работу.

e)                 Поскольку эта величина была постоянной, мы смогли вычислить результат.

Критерии оценки:

5 «отлично» -  Правильное выполнение заданий. Студент отлично знает и применяет на практике грамматический материал по теме Причастие, его формы, отличие причастия от герундия. Студент владеет необходимой  лексикой по теме Оборудование, работа, обладает навыками чтения и литературного перевода.

4 «хорошо» - Правильное выполнение 8 заданий из 9. Студент хорошо знает и применяет на практике грамматический материал по теме Причастие, его формы, отличие причастия от герундия. Студент владеет необходимой  лексикой по теме Оборудование, работа, обладает навыками чтения и литературного перевода. Допускаются неточности при выполнении заданий.

3 «удовлетворительно» - Допускаются неточности и незначительные ошибки в выполнении  заданий. Студент не достаточно хорошо знает и применяет на практике грамматический материал по теме Причастие, его формы, отличие причастия от герундия. Студент не в полной мере владеет необходимой  лексикой по теме Оборудование, работа, обладает навыками чтения и литературного перевода. Объем правильно выполненных заданий составляет 6 заданий.

2 «неудовлетворительно» - Отсутствует выполнение заданий. Студент не знает и не применяет на практике грамматический материал по теме Причастие, его формы, отличие причастия от герундия. Студент не владеет необходимой  лексикой по теме Оборудование, работа, не обладает навыками чтения и литературного перевода. Объем правильно выполненных заданий составляет менее 4 заданий из 9



Приложение Б (обязательное).   Задания для проверки теоретических знаний и практических умений: З1; У1; У2; У3 для рубежного контроля (по окончании 3 семестра). Задания для проверки теоретических знаний и практических умений: З1; У1; У2; У3 для итогового занятия (по окончании 4 семестра).

Задания для проверки теоретических знаний и практических умений: З1; У1; У2; У3 для рубежного контроля (по окончании 3 семестра).

Задание 1. Раскройте скобки, поставьте глагол в Present Simple, Present Continuous или Present Perfect.

1.                 Every day Julie (take) the bus to go to her office.

2.                 Julie usually (clean) the house on Saturdays.

3.                 She (write) a letter to a client now.

4.                  Doctors (cure) many deadly diseases.

5.                 Scientists (split) the atom.

Задание 2. Раскройте скобки, выберите нужное время глагола.

1.                 He (know) several foreign languages.

2.                 He often (visit) them last year.

3.                 She (work) abroad next year.

4.                 At present he (not work) at school.

5.                 I (write) the letter when he came.

Задание 3. Раскройте скобки, поставьте глаголы в нужные формы страдательного залога.

1.                 Those books (return) to the library yesterday.

2.                 The patient (take) to the hospital today and (operate) tomorrow morning.

3.                 This room (use) only on special occasions.

4.                 Usually the experiments (carry out) every day, but they (not carry out) yesterday.

5.                 The problem (discuss) when I came in.

Задание 4. Укажите предложения, в которых ing-форма является причастием, герундием, отглагольным существительным.

1.                 Wishing to learn to skate, she bought herself a pair of skates.

2.                 The children were tired of running.

3.                 My greatest pleasure is travelling.

4.                 Entering the room, I saw my friends smiling at me.

5.                 The singing of those folk songs impressed me greatly.

Задание 5. Дополните предложения, подставив подходящие союзы (both…and, eitheror, neithernor)

1.                 You can do it_______ today or tomorrow.

2.                 I believe _______ him nor his wife.

3.                 They were_______ hungry and tired when they got to the hotel.

4.                 We can develop individual educational program_______ for you nor for your friend.

5.                 You should answer_______  this question or that one.

Задание 6. Заполните пропуски в предложениях соответствующими модальными глаголами (must, should, would, ought to, have to, neednt, can, could, may).

1.                 They …… not do this work themselves.

2.                 You …… take my dictionary.

3.                 You don’t look well, you …… consult the doctor.

4.                 …… you tell me the time?

5.                 My friend lives a long way from his college and …… get up early.

Задание 7. Дополните предложения, подставив подходящие союзы (for, as, till, althoughuntil).

1.                 I can translate this text, …… it is not difficult.

2.                 I shall not rest …...  I have finished my work.

3.                 Problems cannot be solved …… they are accurately defined.

4.                 …… there is no generally accepted theory, there is much data.

5.                 Facts do not cease to exist …... they are ignored.

Задание 8. Укажите номера сложносочинённых и сложноподчинённых предложений.

1.                 He could not come to the lecture because he was ill.

2.                 I found the magazine where I had left it.

3.                 We move to a new flat, that’s settled.

4.                 They went for a walk after they had finished their work.

5.                 It is always easy to begin a war, but it is very difficult to stop one.

Задание 9. Вставьте нужную форму глагола to be, либо have/has.

1.                 He …. born in 1985.

2.                 We …. students now.

3.                 It …. an interesting book.

4.                 He …. no English books.

5.                 I …. no watch.

Задание 10. Перефразируйте предложения, употребив сослагательное наклонение после глагола wish.

Model: I can’t go to the party, but I’d like to.  – I wish I could go to the party.

1.                 I don’t speak French, but I’ll need it at college.

2.                 I have to get up early, but I don’t want to.

3.                 I want to phone him, but I don’t know his number.

4.                 It’s a pity that you weren’t there.

5.                 It’s a pity he didn’t see this film.

Задание 11. Прочитайте текст, выполните задания, следующие после него.

Informal and formal education

Education includes different kinds of learning experiences. In its broadest sense, we consider education to be the ways in which people learn skills, gain knowledge and understanding about the world and about themselves. One useful scheme for discussing education is to divide these ways of learning into two types: informal and formal.

Informal education involves people in learning during their daily life. For example, children learn their language simply by hearing and trying to speak themselves .In the same informal manner, they learn to dress themselves, to eat with good manners, to ride a bicycle, or to make a telephone call. Education  is also informal when people try to get information or to learn skills on their own initiative without a teacher. To do so, they may visit a book shop,library or museum.  They may watch a television show, look at a  videotape, or listen to a radio program. They do not have to pass tests or exams.

We consider formal education to be the instruction given at different kinds of schools, colleges, universities. In most countries, people  enter a system of formal education during their early childhood. In this type of education, the people, who are in charge of education, decide what to teach. Then learners study those things with the teachers at the head. Teachers expect learners to come to school

Regularly and on time, to work at about the same speed as their classmates, and to pass tests and exams. Learners have to pass the exams to show how well they have progressed in their learning. At the end of their learning, learners may earn a diploma, a certificate, or a degree as a mark of their success over the years.

The school systems of all modern nations provide both general and vocational education. Most countries also offer special education programs for gifted or for physically or mentally handicapped children. Adult education programs are provided for people who wish to take up their education after leaving school. Most countries spend a large amount of time and money for formal education of their citizens.

General and vocational education

General education aims at producing intelligent, responsible, well-informed citizens. It is designed to transmit a common cultural heritage rather than to develop trained specialists.

Almost all elementary education is general education. In every country, primary school pupils are taught skills they will use throughout their life, such as reading, writing, and arithmetic. They also receive instruction in different subjects, including geography, history, etc. In most countries almost all young people continue their general education in secondary schools.

The aim of vocational education is primarily to prepare the students for a job. Some secondary schools specialize in vocational programs. Technical schools are vocational secondary schools, where students are taught more technical subjects, such as carpentry, metalwork, and electronics. Technical school students are required to take  some general education  courses and vocational training. Universities and separate professional schools prepare students for careers in such fields as agriculture, architecture, business, engineering, law, medicine, music, teaching, etc.

Подберите к данным словам и выражениям из текста соответствующие русские эквиваленты.

1) in its broadest sense, 2) learn skills, 3) gain knowledge about themselves, 4) useful scheme, 5) formal, 6) informal, 7) on their own initiative, 8) early childhood, 9) at about the same speed, 10) in charge of, 11) gifted children, 12) physically or mentally handicapped children, 13) citizens, 14) general education, 15) vocational education, 16) aims at, 17) vocational training, 18) law, 19) carpentry.

a) получают знания о себе, b) в широком смысле, c) юриспруденция, d) обучаются навыкам, e) полезная схема, f) одаренные дети, g) граждане, h) дети с физическими или умственными недостатками, i) официальный, j) неофициальный, k) в ответственности, l) общее образование, m) по их собственной инициативе, n) профессиональное образование, o) ставит целью, p) раннее детство, q) профессиональная подготовка, r) с приблизительно одинаковой скоростью,  s) плотничное дело.

Ответьте на вопросы по содержанию текста.

1)                What is education?

2)                What is informal education?

3)                What is formal education?

4)                What are different kinds of formal education? 

5)                What are pupils in primary school taught?

6)                What is the main aim of vocational education?

7)                What are technical schools?

8)                What profession can students at University get?

КРИТЕРИИ ОЦЕНКИ:

5 «отлично» -  Правильное выполнение заданий. Студент отлично знает и применяет на практике грамматический материал по темам: времена английского глагола, формы страдательного залога, ing-формы (причастие1, герундий, отглагольное существительное), союзы both…and, eitheror, neithernor, модальные глаголы,  союзы for, as, till, althoughuntil, сложносочинённые и сложноподчинённые предложения, формы глагола to be, to have, сослагательное наклонение после глагола wish. Студент владеет необходимой  лексикой по теме: Education.

4 «хорошо» - Правильное выполнение 8-9 заданий из 11. Студент хорошо знает и применяет на практике грамматический материал по темам: времена английского глагола, формы страдательного залога, ing-формы (причастие1, герундий, отглагольное существительное), союзы both…and, eitheror, neithernor, модальные глаголы,  союзы for, as, till, althoughuntil, сложносочинённые и сложноподчинённые предложения, формы глагола to be, to have, сослагательное наклонение после глагола wish. Студент владеет необходимой  лексикой по теме: Education. Допускаются неточности при выполнении заданий.

3 «удовлетворительно» - Допускаются неточности и незначительные ошибки в выполнении  заданий. Студент знает и применяет на практике грамматический материал по темам: времена английского глагола, формы страдательного залога, ing-формы (причастие1, герундий, отглагольное существительное), союзы both…and, eitheror, neithernor, модальные глаголы,  союзы for, as, till, althoughuntil, сложносочинённые и сложноподчинённые предложения, формы глагола to be, to have, сослагательное наклонение после глагола wish. Студент не достаточно хорошо владеет лексикой по теме: Education.

  Правильно выполнено 4-7 заданий.

2 «неудовлетворительно» - Отсутствует выполнение заданий. Студент не знает и не применяет на практике грамматический материал по темам: времена английского глагола, формы страдательного залога, ing-формы (причастие1, герундий, отглагольное существительное), союзы both…and, eitheror, neithernor, модальные глаголы,  союзы for, as, till, althoughuntil, сложносочинённые и сложноподчинённые предложения, формы глагола to be, to have, сослагательное наклонение после глагола wish. Студент не владеет необходимой  лексикой по теме: Education.

 Правильно выполнено менее 4 заданий из 11.

 

 

 

 

 

 

 

 

 

 

 

 

 

 

 

 

 

 

 

 

 

 

 

 

 

 

 

 

 

Задания для проверки теоретических знаний и практических умений: З1; У1; У2; У3 для итогового занятия (по окончании 4 семестра)

Задание 1. Перефразируйте предложения, употребив сослагательное наклонение после глагола wish.

Model: I am sorry. I don’t know Mr. Smith. I wish I knew Mr. Smith.

1.                 I am sorry I cannot do this.

2.                 I am sorry you cannot read this book in the original.

3.                 I am sorry the play isn’t a success.

4.                 I am sorry I cannot change it.

5.                 I am sorry you are not healthy.

Задание 2. Образуйте вежливую и отрицательную формы повелительного наклонения.

E.g. Do it now. Will you do it now?/ Do it now please. Don’t do it now.

1.                 Check the test.

2.                 Fix the tent.

3.                 Let us risk.

4.                 Shift the block.

5.                 Test the net.

Задание 3. Раскрывая скобки, напишите каждое предложение три раза, образуя условные предложения 1, 2 и 3 типов.

E.g. If you (to be) free, I (to come) to see you.

If you are free, I shall come to see you.

If you were free, I should come to see you.

If you had been free, I should have come to see you.

1.                 If you (to be) busy, I (to leave) you alone.

2.                 If my mother (to buy) a cake, we (to have) a very nice tea party.

3.                 If I (to live) in Moscow, I (to visit) the Tretyakov Art Gallery every year.

4.                 If she (to know) English, she (to try) to enter the university.

5.                 If I (to get) a ticket, I (to go) to the Philharmonic.

Задание 4. Дополните предложения, подставив подходящий по смыслу инфинитив.

(to eat, to trust, to brush up, to speak, to interrupt)

1.                 …….. you all the time was very rude.

2.                 I’d like ……... to the dean.

Задание 5. Укажите номера предложений, содержащих инфинитивный оборот Сложное дополнение и номера предложений, содержащих инфинитивный оборот Сложное подлежащее.

1.                 Boris is said to have been to London several times.

2.                 The British National Health Service is believed to be the best health care system in the world.

3.                 I want you to speak to your manager.

4.                 I consider Pushkin to be the greatest Russian poet.

5.                 I heard him cough.

Задание 6. Дополните предложения, подставив подходящий по смыслу герундий.

(smoking, surfing, getting up, eating, working)

1.                 ……… with this actor is always a pleasure for film directors.

2.                 ……… early is not much fun.

3.                 ……… fruit and vegetables is healthy.

4.                 Her only hobby is ……… the net.

5.                 ……….  on board the plane is prohibited.

Задание 7. Переведите предложения, содержащие герундиальные обороты.

1.                 We knew of his having read his report at the conference.

2.                 We know of the earth behaving as a large magnet.

3.                 We knew of Newton’s having written “The Principia” in a very short time.

4.                 I don’t mind his opening the window.

5.                 She insisted on her son being allowed to go home.

Задание 8. Определите функцию причастия 1 в каждом предложении (функция определения, функция обстоятельства).

1.                 The crying girl is my friends’ daughter.

2.                 Reading a newspaper, she fell asleep.

3.                 She left the room laughing.

4.                 Jim broke his leg playing football.

5.                 He looked at the man sitting in front of him.

Задание 9. Выберите из скобок требующуюся форму причастия.

1.                 The girl (writing, written) on the blackboard is our best student.

2.                 Everything (writing, written) here is quite right.

3.                 Who is that boy (doing, done) his homework?

4.                 The exercises (doing, done) by the pupils were easy.

5.                 Read the (translating, translated) sentences once more.

Задание 10. Прочитайте текст, выполните следующие задания.

Подберите к данным словам и выражениям из текста соответствующие русские эквиваленты.

footwear                                           хлопок-сырец

dominant branch                               хлопчатобумажное производство

cotton textiles                                   обувь

raw cotton                                        преобладающая отрасль

durable consumer goods          отрасли промышленности, требующие     квалифицированной рабочей силы

and the like                                       товары длительного пользования

washing machines                             преимущественно

primarily                                          и тому подобное

skilled industry                                 стиральные машины

Перепишите текст, заполните пропуски в тексте подходящими по смыслу словами, данными ниже.

Light Industry

Russia’s textile industries are heavily concentrated in the European sector, especially in the Central region, which produces a large share of the federation’s clothing and________.

The ________ is ________, with the ________ coming mainly from the Central Asian states. In the zone between the Volga and Oka rivers, east of Moscow, there are numerous cotton textile towns, the largest of which are Ivanovo, Kostroma, and Yaroslavl. ________– refrigerators, ________, radios and television sets, and the like – are produced________ in areas with a tradition of ________, notably in and around Moscow and St.Petersburg.

Primarily, raw cotton, footwear, dominant branch, durable consumer goods, skilled industry, cotton textiles, washing machines

Ответьте на вопросы к тексту.

1)                Where areRussia’s textile industries heavily concentrated?

2)                What is the dominant branch oftextile industry?

3)                What are the largestcotton textile towns in the zone between the Volga and Oka rivers?

4)                Where are durable consumer goods primarilyproduced?

5)                Does raw cotton comemainly from the Central Asian states?

КРИТЕРИИ ОЦЕНКИ:

5 «отлично» -  Правильное выполнение заданий. Студент отлично знает и применяет на практике грамматический материал по темам: сослагательное и повелительное наклонение в английском языке, условные предложения различных типов, инфинитив, инфинитивный оборот Сложное дополнение ComplexObject, инфинитивный оборот Сложное подлежащее Complex Subject, герундий и герундиальные обороты, функции и формы причастия.

Студент владеет необходимой  лексикой по темe: Light Industry.

4 «хорошо» - Правильное выполнение 7-9 заданий. Студент хорошо знает и применяет на практике грамматический материал по темам: сослагательное наклонение, повелительное наклонение в английском языке, условные предложения различных типов, инфинитив, инфинитивный оборот Сложное дополнение Complex Object, инфинитивный оборот Сложное подлежащее Complex Subject , герундий и герундиальные обороты, функции и формы причастия.

Студент владеет необходимой  лексикой по теме: Light Industry. Допускаются неточности при выполнении заданий.

3 «удовлетворительно» - Допускаются неточности и незначительные ошибки в выполнении  заданий. Студент знает и применяет на практике грамматический материал по темам: сослагательное и повелительное наклонение в английском языке, условные предложения различных типов, инфинитив, инфинитивный оборот Сложное дополнение Complex Object, инфинитивный оборот Сложное подлежащее Complex Subject, герундий и герундиальные обороты, функции и формы причастия.

Студент не достаточно хорошо владеет необходимой  лексикой по теме: Light Industry. Объем правильно выполненных заданий составляет 4-6 заданий.

2 «неудовлетворительно» - Отсутствует выполнение заданий. Студент не знает и не применяет на практике грамматический материал по темам: сослагательное и повелительное наклонение в английском языке, условные предложения различных типов, инфинитив, инфинитивный оборот Сложное дополнение Complex Object, инфинитивный оборот Сложное подлежащее Complex Subject, герундий и герундиальные обороты, функции и формы причастия.

Студент не владеет необходимой  лексикой по теме: Light Industry. Правильно выполнено менее 3 заданий из 10.


 

Скачано с www.znanio.ru

Комитет образования, науки и молодежной политики

Комитет образования, науки и молодежной политики

Комплект оценочных средств разработан на основе рабочей программы учебной дисциплины

Комплект оценочных средств разработан на основе рабочей программы учебной дисциплины

Содержание стр

Содержание стр

Общие положения Результатом освоения учебной дисциплины (УД) на втором курсе являются освоенные умения и усвоенные знания , направленные на формирование общих и профессиональных компетенций

Общие положения Результатом освоения учебной дисциплины (УД) на втором курсе являются освоенные умения и усвоенные знания , направленные на формирование общих и профессиональных компетенций

Раздел 1. Основной раздел

Раздел 1. Основной раздел

Тема 1.1. Образование в России и за рубежом, среднее профессиональное образование

Тема 1.1. Образование в России и за рубежом, среднее профессиональное образование

Глаголы в действительном залоге, в

Глаголы в действительном залоге, в

Охарактеризовать свой типичный рабочий день

Охарактеризовать свой типичный рабочий день

Каковы преимущества и недостатки путешествий с помощью различных видов транспорта? 2

Каковы преимущества и недостатки путешествий с помощью различных видов транспорта? 2

Тема 2.2 Основные геометрические понятия и физические явления

Тема 2.2 Основные геометрические понятия и физические явления

Отличие зависимых причастных оборотов от абсолютных независимых причастных оборотов

Отличие зависимых причастных оборотов от абсолютных независимых причастных оборотов

ОК.7 Брать на себя ответственность за работу членов команды (подчинённых), за результат выполнения заданий

ОК.7 Брать на себя ответственность за работу членов команды (подчинённых), за результат выполнения заданий

Приложение А (обязательное).

Приложение А (обязательное).

Questions: 1. Is education in our country free? 2

Questions: 1. Is education in our country free? 2

After finishing such a school a pupil becomes an unskilled worker

After finishing such a school a pupil becomes an unskilled worker

Распределите слова по группам: a ) неформальное образование, b ) формальное образование, c ) специальное образование

Распределите слова по группам: a ) неформальное образование, b ) формальное образование, c ) специальное образование

At secondary school pupils are taught different _________

At secondary school pupils are taught different _________

Глаголы в действительном залоге, в

Глаголы в действительном залоге, в

Russia [ˈrʌʃə] Россия Political system [ pəˈlɪtɪk(ə)l ˈsɪstəm ]

Russia [ˈrʌʃə] Россия Political system [ pəˈlɪtɪk(ə)l ˈsɪstəm ]

What branches does the country government consist of? 2)

What branches does the country government consist of? 2)

The federal government is the national government of the

The federal government is the national government of the

Работа только что закончена нами

Работа только что закончена нами

Will the final exams be taken in summer or in winter? (Выпускные экзамены будут сдаваться летом или зимой?) 2

Will the final exams be taken in summer or in winter? (Выпускные экзамены будут сдаваться летом или зимой?) 2

Проверяемые результаты обучения: владение навыками чтения и перевода , знание модальных глаголов и их эквивалентов, умение распознавать и употреблять их в речи

Проверяемые результаты обучения: владение навыками чтения и перевода , знание модальных глаголов и их эквивалентов, умение распознавать и употреблять их в речи

When do you usually get up on your working day? 2

When do you usually get up on your working day? 2

Soft skills are no less important than hard ones, and both are better developed already in adolescence

Soft skills are no less important than hard ones, and both are better developed already in adolescence

Я не люблю поздно ложиться спать, но иногда мне приходится

Я не люблю поздно ложиться спать, но иногда мне приходится

Студент владеет необходимой лексикой по теме

Студент владеет необходимой лексикой по теме

Задание 2. Прочитайте текст, ответьте на вопросы на понимание содержания

Задание 2. Прочитайте текст, ответьте на вопросы на понимание содержания

Only a healthy man can enjoy his life, work well and be happy

Only a healthy man can enjoy his life, work well and be happy

Questions: 1. Why do people all over the world are fond of sports and games? 2

Questions: 1. Why do people all over the world are fond of sports and games? 2

Each team can have up to seventeen players, but only six can play at the same time

Each team can have up to seventeen players, but only six can play at the same time

Alice will be picking up the berries

Alice will be picking up the berries

Задания (типовые) для оценки освоения раздела 1, темы 1

Задания (типовые) для оценки освоения раздела 1, темы 1

Задание 2. Прочитайте текст, составьте план и краткое изложение содержания текста

Задание 2. Прочитайте текст, составьте план и краткое изложение содержания текста

Задание 3. Прочитайте следующие утверждения, определите правдивы они или ложны

Задание 3. Прочитайте следующие утверждения, определите правдивы они или ложны

Complete the sentences with the words :

Complete the sentences with the words :

This girl says she can speak three languages

This girl says she can speak three languages

Раздел 1. Тема 1.6 Отдых, каникулы, отпуск

Раздел 1. Тема 1.6 Отдых, каникулы, отпуск

Задание 2. Ответьте на вопросы к тексту

Задание 2. Ответьте на вопросы к тексту

Soviet Union (Russia, Ukraine,

Soviet Union (Russia, Ukraine,

Ecotourism (also known as ecological tourism) is a form of tourism, that appeals to ecologically and socially conscious individuals

Ecotourism (also known as ecological tourism) is a form of tourism, that appeals to ecologically and socially conscious individuals

Their names (10) ______ Gina and

Their names (10) ______ Gina and

Вспомогательные глаголы to be, to have, to do

Вспомогательные глаголы to be, to have, to do

I am allowed — Мне разрешают

I am allowed — Мне разрешают

Every generation is unique in its experience

Every generation is unique in its experience

But if the young people do not like noisy clubs and other places of entertainment , they can find friends without leaving their homes –…

But if the young people do not like noisy clubs and other places of entertainment , they can find friends without leaving their homes –…

A bully is anyone who feels powerful when they hurt or pick on someone else

A bully is anyone who feels powerful when they hurt or pick on someone else

Only weak people can be addicted, can’t they? 2

Only weak people can be addicted, can’t they? 2

If he … (be) in town, he would have been present at our meeting

If he … (be) in town, he would have been present at our meeting
Материалы на данной страницы взяты из открытых истончиков либо размещены пользователем в соответствии с договором-офертой сайта. Вы можете сообщить о нарушении.
13.06.2022